AP Psychology Course Guide
AP Psychology Course Guide
1
AP Psychology Course Syllabus
Objectives
1. Students will prepare for the AP Psychology Exam.
2. Students will develop an understanding of the major concepts, terms, and theories
of psychology.
3. Students will learn the basic skills of psychological research enabling them to devise
research projects, interpret, generalize and validate research reports.
4. Students will develop the ability to think critically.
5. Students will gain an appreciation for the science of Psychology.
6. Students will gain a mutual respect for people and their individual differences,
preparing them for life in society.
7. Students will learn the ethical standards governing the work of psychologists.
Requirements
All students are required:
1. To be prepared daily to participate in class discussions and take notes.
2. To be current on all readings assigned.
3. To complete homework as assigned including vocabulary, which must be completed
prior to each unit test.
4. To submit a research paper on a topic of the student’s choosing.
5. To submit a portfolio during the fourth marking period.
Grading
All grades will be calculated using straight percentages. Probable weights:
Tests 100 points each
Quizzes 20 – 25 points each
Free Responses 20 – 25 each
Homework 10 points each
Research Paper 200 points
Portfolio 200 points
*Midterm/Final Exam 150 points/150 points
*The midterm exam will be administered January and will encompass all content covered to that point; the
rest of the content will be assessed at the conclusion of the course. *
Conduct
Students will follow all class rules as they apply to the EAHS student handbook. Missed
work is the student’s responsibility to make up. Advanced assignments (research paper
and portfolio) must be turned in on time with no exceptions! If absent on the day an
assignment (homework, journal, etc.) is due, it must be turned in on the day the student
returns. The assignment will only be accepted providing the absence was excused.
2
Topic Outline
1. Methods, Approaches, History (2 weeks)
2. Biological Basis of Behavior (3 weeks)
3. Sensation and Perception (3 weeks)
4. States of Consciousness (Independent study)
5. Learning (2 weeks)
6. Memory & Cognition (3weeks)
7. Motivation and Emotion (2 weeks)
8. Developmental Psychology (2 weeks)
9. Personality (2 weeks)
10. Testing and Individual Differences (2 weeks)
11. Abnormal Psychology & Treatment (3 weeks)
12. Social Psychology (2 weeks)
**The above topics are taken from the Advanced Placement Course Description**
Textbook
• Psychology, AP Edition (2009) by Zimbardo, Johnson, & Weber
Additional Resources
• Moodle: http://learn.eastonsd.org/
Possible Films
During the course of the year, it is possible that we will watch either a segment or the
entirety of the following films in order to enhance your understanding of the given topic.
• A Beautiful Mind (2001) Rated PG 13 (Abnormal Psych & Treatment)
• Inside Out(2015) Rated PG (Emotions & Personality)
• The Hunger Games (2012) Rated PG-13 (Social Psychology)
• Inception (2010) Rated PG-13 (Consciousness)
Please complete the information found below, which signifies your acknowledgement and comprehension of
the objectives, conduct expectations, and grading procedures. Also, your signature grants your child
permission to watch segments of, or the entire films, listed above.
________________________________________________________________________________________________
Student Name/Signature
________________________________________________________________________________________________
Parent Name/Signature
Should you have any questions or concerns, please feel free to contact me at [email protected]
3
AP Psychology Course Outline
4
Unit VII - Motivation and Emotion (2 weeks)
A. Theories of Motivation: Instinct, Arousal, Drive, Incentive, Cognitive and
B. Maslow’s Hierarchy
C. Specific Motivations: Hunger, Work, Sexual, Aggressive
D. Biological basis of Emotions
E. Theories of Emotion: James-Lange, Canon-Bard, Two Factor, Opponent-Process.
F. Causes, Effects, and Control of Stress, G.A.S. Model
Unit IX – Human Development I (3 weeks)
A. Longitudinal and Cross-Sectional Studies
B. Prenatal Period and Influences on Development
C. Cognitive Development: Piaget and Vygotsky
D. Moral Development: Kohlberg
E. Social and Emotional Development: Erikson
F. Death and Bereavement: Elisabeth Kubler-Ross
Unit X – Personality (2 weeks)
A. Personality Theory: Freud, Jung, Adler, Rogers
B. Trait Theory: Big 5 Factors, Nomothetic Approach, Idiographic Approach
C. Self concept, Self-esteem, Self-Reinforcement
D. Personality Assessment
Unit XI – Intelligence/Individual Differences & Testing (2 weeks)
A. Achievement and Intelligence
B. Theories of Intelligence: Spearman, Thurstone, Gardner, Sternberg, Goleman
C. Psychological Testing: History, Reliability, Validity, Central Tendency, Standard
Deviation
Unit XII – Mental Disorders and Treatment (3 weeks)
A. Defining Abnormality: Medical Model, Deviance, Personal Distress, Maladaptive
Behavior
B. Diagnostic Labeling: Rosenhan Study
C. Classification of Disorders: DSM V
D. Categories of Disorders: Anxiety, Mood, Dissociative, Somatoform, Personality,
Schizophrenia
E. Major Therapies: Psychotherapy, Humanistic, Client- centered, Rational-
Emotive, Cognitive, Group.
Unit XIII – Social Thought and Social Behavior (2 weeks)
A. Attitudes and Behavior: Attribution, Roles, Cognitive Dissonance, Self Serving
Bias, Actor-Observer Bias
B. Conformity, Obedience, and Compliance: Asch Experiment, Milgram Studies,
Stanford Prison Experiment
C. Prejudice and Scapegoating
D. Pro-social Behavior: Bystander Apathy and Bystander Intervention
E. Group Behavior: Anonymity, Unanimity, De-individuation, Social Loafing
5
Research Paper Assignment
Each paper should be 5 - 6 pages typed, double-spaced with a font size of 12, not counting
the cover page, abstract, and references page. Your research should include a minimum of
4 references, with at least 2 references coming from databases searches (ex. EBSCOhost).
Print the abstracts of your chosen journal articles (an abstract is a brief summary of the
article that appears at the beginning of its write up) and attach to your reference page.
To find you’re your articles, you may utilize the library’s website
http://destiny.eastonsd.org/common/servlet/presenthomeform.do?l2m=Home&tm=Hom
e&l2m=Home. You may access the databases from the school or at home (with an Easton
Area Public Library card). If you have any questions about the appropriateness of your
articles or are having trouble finding what you want, please check with me.
Your research papers are to be prepared in APA format, and as such should include:
APA Format for References (fictitious samples, to show you the format):
Please utilize the following website as a reference:
http://owl.english.purdue.edu/owl/resource/560/01/
• For a book:
• Example:
o Erikson, E. (1958). Psychological and social development through the lifespan.
(pp. 104-155). New York, NY: Hardcourt Brace.
6
Your paper should address the following:
Grading:
1. Please refer to the SAMPLE rubric that follows the assignment
2. This research paper is worth 200 points
3. Additional grades will be given during the 1st and 2nd quarters for progress
check-ins
4. Failure to hand this research paper in by the due date will result in a deduction
of one letter grade per day. Weekends and holidays count as a day. A
maximum of three days lateness will be allowed. All other late papers will
result in an automatic zero.
Topic List:
7
11. The Validity of Psychological Tests: how established? Which tests more valid? Less
valid?
12. Collectivist vs. Individualistic Cultures: impact on positive self-esteem/sense of
self.
13. Interpersonal Attraction: causes, gender differences; is there an "ideal"?
14. Gender differences and the Brain - separating actual differences from stereotypes
15. Endorphins and Happiness: production, role, implications for mental health
16. Learned Optimism/helplessness: how each come about; implications for mental
health
17. Genius: Criteria, Characteristics and Types.
18. The Internet: positive/negative effects on our psychological health
19. How has Abraham Maslow's concept of self-actualization affected Psychology?
20. Childhood Trauma: impact on personality/role in personality disorders
21. Race and Intelligence: The Scientific View
22. Possible Causes of Childhood Autism
23. How is Intelligence Measured? Reasons for controversy/debate
24. The Dual Roles of Genetics and Environment on Intelligence
25. Inherited Behavior - The Latest Research
26. Alzheimer's Disease: causes, treatment & role lifestyle plays
27. Contact sports & degenerative brain disease
28. Workplace stress: environmental factors? Role of culture, of job type?
29. Rational Emotive Theory: development & modern uses
30. Aging: changes to brain; implications, opportunities & challenges
31. Theories of Forgetting - explanation & implications for better memory
32. Does money buy happiness? Psychological health & socioeconomic status
33. Theories of Intelligence (Spearman vs. Gardner): implications for schools
34. The Theory of Persuasion - why works/doesn't work (psychology behind it)
35. Bullying: causes, who gets bullied; what can parents/teachers do to stop it?
36. The Effects of Over Crowding on Human Beings
37. Subliminal Messages and How They can be Used to Influence Thought and Behavior
38. Current Experiments in Thought Control
39. The Psychological Effects of Torture
40. How are Children Affected by War.
41. The Theory of Memory Storage in the Human Brain: is there a physical basis? What
happens to it?
42. How to Motivate Children to Achieve
43. The Function of Play in psychological health/well-being
44. Reality Therapy: origins, modern uses
45. The Psychological Benefits of Pets & Pet Therapy
46. Psychological Problems of Paraplegics
47. The Emotional Effects of Solitary Confinement
48. Behavior Patterns of Hyperactive Children - biological vs. environmental roots
49. The Stages of Grief According to Elizabeth Kubler-Ross: implications for coping with
8
loss
50. Attractiveness & bias: are the attractive treated differently? (convictions, hiring,
perceptions)
51. Habits: How are they Formed and How They Can Be Changed?
52. The Effects of Color on Mental States
53. The Idiot Savant: origins; contrast with normal intellectual functioning
54. The Theory of Boredom: mental/emotional determinants
55. Managing Interpersonal Conflict: psychology of mediation
56. Divorce: causes, psychological impact
57. Shyness in Adults: causes, consequences
58. How Children Develop Gender Identification
59. Typical Mother-Daughter Conflicts in Modern America: causes, solutions
60. The Psychological Aspects of Obesity
61. The Psychological Profile of a Terrorist
62. The Clinical Uses of Hypnosis - risks vs. benefits
63. Significant Findings From Recent Research on Identical Twins
64. The Psychological Profile of a Mercenary Soldier
65. Abusers: A Profile - early psychological history of a future abuser
66. Eating Disorders: why are they so resistant to treatment? Role of gender? Brain
changes?
67. Hate Crimes: impact on victims & on communities
68. Dreams & dream interpretation: why they occur? are interpretations valid?
69. The Limits of Artificial Intelligence
70. Compare and Contrast the Training, Scope, and Functions of Psychiatrists and
Psychologists.
71. The Rorschach Test: How is it Administered and evaluated?
72. The Psychological Problems of the Homeless.
73. How does social anxiety disorder affect the routine life of a person?
74. Cyber bullying: psychological impact and recommended responses
75. ADHD: Latest research and recommended treatment
76. The link between exercise and emotional health
77. Anger management: what makes psychological treatment effective?
78. Psychology of Flow: implications for workplace satisfaction
79. Psychological characteristics of long-lasting marriages
80. Creativity and Intelligence: what makes kids creative, and is it measured by
intelligence tests?
81. US Cultural and Marketing Trends: do they threaten children's physical and mental
health?
82. Violence in the Media: psychological effects on children
83. Tools of Sports Psychologists: how they can also promote mental health in the
workplace
9
PORTFOLIO ASSIGNMENT
Purpose: This assignment is designed to encourage the student to monitor and self-
assess as a part of the learning experience in the Advanced Placement Psychology course.
Students should be encouraged to include psychology assignments and projects (journals,
letter to a psychologist, etc.)
Overview: This assignment can generate a good deal of interest and involvement on the
part of the student’s parents, other family members, and friends. Students are encouraged
to begin this project early in the AP Psychology course.
You are required to create a book about yourself. Essentially, you are conducting a case
study about you.
I believe that this book will stimulate reflections on your life, the people and events in it,
and the future. I also believe that it may be a source of inspiration and motivation. Most
importantly, I believe that it will help you in achieving a greater sense of your own identity
and personal worth. Finally, it is hoped that you will examine your life in terms of the
context of the Psychology class. Students who have done this project in the past have found
that preparing this assignment was a truly worthwhile exercise; it stimulated personal
growth, gave them a greater sense of their individual identities, and helped them become
more appreciative of many of the people who have contributed significantly to their lives.
If you believe that there are some special circumstances that make it impossible for you to do
this project, you must talk to me about this within the next few days.
PRIVACY:
Due to the nature of this assignment, you have the right to be concerned with your privacy.
Although I establish the guidelines, you are in control of the details. You determine the
specifics. If you have concerns about this, be sure to talk to me about them. You do not
have to show your portfolio to anyone else if you do not want to.
THE BOOK:
This project does not need to be a costly one. It is entirely possible to create a digital copy
of your portfolio by scanning pictures, including clip art and headlines, etc. I ask that if
you create your portfolio digitally, that you print a hard copy and turn it in a binder.
10
If you choose to create a traditional portfolio, procure items that are functional and durable
(scrapbooks, 3-ringer binders, plastic protective sheets, etc.)
LETTER GRADES:
Even though the greatest value in this project is not the grade that you receive, you will
indeed receive a letter grade for your accomplishment. The following criteria will be
evaluated in order to determine your grade:
1. Completeness: your book meets all content requirements. There will be a one
letter grade reduction for each section missing or incomplete.
2. Following Instructions: This includes labeling, identifying, explaining, and
following organization.
3. Appearance: It is neat, attractive, and demonstrates creativity. Written material
generated by you must be typed in a 12-point font (Arial, Times New Roman, and
Cambria are acceptable fonts).
4. Essays: all required essays are included and are well written. They reflect a
serious or thoughtful approach to each topic assigned. (See the next section for the
list of required essays.)
1. Table of Contents
2. Personal Resume and Introduction
3. My Favorites (from each stage of development if possible)
4. Genealogy (optional)
5. Interview (or letter) of/from your parents about you (from birth to present)
6. My Family (2 pages)
7. My Friends (2 pages)
8. Activities: School, Church, and Community (optional)
9. Awards/Recognition/Honors Received (optional)
10. Religious/Spiritual/Motivational/Points to Ponder (3 pages)
*This section may include appropriate poetry, lyrics, writings, quotations,
pictures, etc.
11. Beauty (pictures, photos, poems (2 pages)
*No pictures of your “dream” girl/guy. Again, this section may include
appropriate poetry, lyrics, writings, quotations, pictures, etc.
12. Mementoes/Memorabilia (2 pages)
*This section may include concert tickets, movie stubs, stickers, pictures, etc.
13. Humor (2 pages)
*Include appropriate materials that make you laugh hysterically.
11
Jokes, cartoons, comic strips, anecdotes, etc.
14. Essays: must be typed in 12-font and double-spaced. All of the following essays are
required. As you write these essays, think about the theories we studies in
Psychology class. At this time, has this course, and particularly this assignment
changed anything about you?
A. “Memorable Events in My Life” (2 pages)
B. “Special People in My Life” (2 pages)
COMMENTS:
This project will take many hours of work to do it properly, but it will also be enjoyable
once you get into it. I also believe that it will be one of the most meaningful projects or
assignments you did throughout your school years. If you put yourself into it and do it
properly, it will be the source of a great deal of pride in yourself and your product.
Get started today!! Procrastination or delay may very well put you in a serious bind as the
deadline draws near.
12
Re-read these pages at different times to avoid errors, omissions, and just to keep yourself
on track.
Please feel free to consult with me on ANY questions, concerns, or problems you may have
or that may arise later on.
1. Behavioral Perspective
2. Behaviorism
4. Cognitive Perspective
5. Developmental Perspective
6. Evolutionary Perspective
7. Functionalism
8. Humanist Perspective
13
10. Introspection
14. Structuralism
20. Correlation
14
24. Hawthorne Effect
26. Hypothesis
32. Population
35. Sampling
15
38. Theory
16
E. environmentally determined.
6. A researcher studies the effects of brain lesions in rats. The research study reflects
which of the following perspectives?
A. Cognitive
B. Biological
C. Behavioral
D. Humanistic
E. Psychoanalytic
A student hypothesizes that high school students consuming different flavors of a drink
before a spelling test will perform differently. A study to test the hypothesis finds that with
a bitter drink, performance is best 6 hours after drinking it, whereas with a sweet drink,
performance is better 1 hour after drinking it.
10. Dr. Lewis decided to study the television viewing habits of her students and therefore
demanded that they complete and return a survey regarding their viewing habits. Which of
the following ethical principles was clearly violated in this study?
A. The right to have a copy of the results
17
B. The right to a copy of the survey
C. The right to be protected from unnecessary risk of harm
D. The right to refuse to participate in the study
E. The right to be informed of deception
11. A researcher conducts an experiment to test the claim that new drug Y is more effective
than standard drug X in inhibiting arousal. The researcher randomly assigns participants
to receive drug Y or drug X and subsequently measures arousal. In this experiment,
participants who receive drug Y constitute the
A. dependent variable
B. confounding variable
C. experimental group
D. control group
E. placebo group
12. A market researcher is interested in ascertaining which of two possible packages is
more likely to entice buyers to purchase a new brand of chocolate-chip cookies. In this
study, which of the following pairs represents the independent and dependent variables,
respectively.
A. Another brand of cookie; the new brand of cookie
B. The new brand of cookie; another brand of cookie
C. The new brand of cookie; the two types of packaging
D. Total sales for each of the packages; the different types of package
E. The different types of packages; total sales for each of the packages
39. Acetylcholine
42. Agonist
44. Amygdala
45. Antagonist
47. Axon
19
48. Brain Lateralization
51. Cerebellum
55. Dendrites
56. Dopamine
60. Endorphins
20
62. Functional MRI (fMRI)
63. Hippocampus
64. Hypothalamus
68. Medulla
71. Neuron
72. Neurotransmitters
21
76. Parietal lobe
79. Plasticity
80. Pons
84. Serotonin
87. Synapse
22
90. Thalamus
23
e. Electrical impulse
6. The human brain differs from the brains of most other animals by the relative amount
of brain mass devoted to which of the following?
a. The occipital lobe
b. The cerebellum
c. The cerebral cortex
d. The homunculus
e. Pituitary gland
7. Which of the following structures of the brain has been linked with the regulation of
hunger and thirst?
a. Hippocampus
b. Hypothalamus
c. Thalamus
d. Pons
e. Medulla
8. Prozac functions as an antidepressant medication because it
a. Enhances production of acetylcholine
b. Blocks the reuptake of the neurotransmitter serotonin
c. Causes selective memory loss for depression-producing events
d. Produces a steady, mild state of euphoria
e. Inhibits frontal lobe activity related to depression
9. Which of the following neurotransmitters is most directly associated with Alzheimer’s
disease?
a. Dopamine
b. Serotonin
c. GABA
d. Acetylcholine
e. Glutamate
10. A bundle of nerve fibers that interconnects the two brain hemispheres is the
a. Association areas.
b. Thalamus.
c. Corpus callosum.
d. Sylvia fissure.
e. Optic chiasm.
11. Which gland of the endocrine system controls the activity of other glands?
a. Pancreas
b. Thyroid
c. Adrenal
d. Pituitary
e. Pineal
24
12. Stimulation of the sympathetic nervous system most frequently results in which of the
following?
a. Acceleration of heart and breathing rates and dilation of pupils
b. Stimulation of tearing, salivation, and peristalsis
c. Calming of the body
d. Contraction of the muscles of the hands and feet
e. Raising the perceptual threshold of olfactory and gustatory receptors
13. In neurons, neurotransmitters are released at the
a. axons. d. receptor sites.
b. cell bodies. e. synaptic vesicles.
c. dendrites.
14. In an emergency situation, the adrenal glands secrete hormones that cause all of the
following EXCEPT
a. Increased heart rate.
b. Increased blood pressure.
c. Increased blood sugar level.
d. Reduced mood swings.
e. Reduced flow to the digestive system.
15. Which of the following is a hormone that is most directly related to human sex drive?
a. Acetylcholine
b. Testosterone
c. Angiotensin
d. Dopamine
e. Norepinephrine
16. Computer-generated color images of the brain that provide information about glucose
metabolism are produced by which of the following techniquest?
a. EEG (Electroencephalography)
b. EMG (Electromyography)
c. MRI (Magnetic Resonance Imaging)
d. PET (Positron Emission Tomography)
e. CAT (Computerized Axial Tomography)
25
Unit 3: Sensation & Perception
99. Cochlea
26
102. Cones
104. Convergence
105. Cornea
110. Fovea
113. Habituation
114. Interposition
27
116. Law of common fate
126. Perception
128. Photoreceptors
28
130. Pupil
131. Retina
133. Rods
134. Sensation
140. Transduction
29
144. Weber’s Law
1. The longer an individual is exposed to a strong odor, the less aware of the odor the
individual becomes. This phenomenon is known as sensory
a. Acuity d. reception.
b. Adaptation e. overload.
c. Awareness
2. The ability to choose specific stimuli to learn about, while filtering out or ignoring other
information, is called
a. Selective attention d. masking.
b. Subliminal perception e. shadowing.
c. Time-sharing
3. The human vestibular sense is most closely associated with the
a. Skin d. olfactory bulb.
b. Semicircular canals e. rods and cones.
c. Taste buds
4. Which of the following is a possible reason why cats see better at night than can
humans?
a. Cats have higher proportion of rods to cones
b. Cats’ pupils can contract to a smaller opening
c. Cats have a smaller blind spot
d. Cats have a large optic nerve tract
e. The visual cortex of cats is located farther forward in the cortex
5. The minimum intensity at which a stimulus can be detected at least 50 percent of the
time is known as the
a. Visual cliff d. receptor potential.
b. Just noticeable difference e. absolute threshold.
c. Perceptual set
6. When Jason practices the drums, he tends not to hear the phone. Today he is expecting
a call from a record producer and answers the phone each time it rings even when he is
practicing the drums. Which of the following explains why Jason hears the phone
today?
a. Weber’s Law
b. Accommodation
c. Frequency Theory
30
d. Signal Detection Theory
e. Harmonics
7. Which of the following is NOT a Gestalt principle of perceptual organization?
a. Proximity d. Intensity
b. Similarity e. Continuty
c. Closure
8. Which of the following is the correct sequence of anatomical structures through which
an auditory stimulus passes before it is perceived as a sound?
a. Cochlea, ossicles, eardrum, oval window, auditory canal
b. Eardrum, cochlea, auditory canal, ossicles, oval window
c. Oval window, auditory canal, eardrum, cochlea, ossicles
d. Ossicles, eardrum, cochlea, auditory canal, oval window
e. Auditory canal, eardrum, ossicles, oval window, cochlea
9. Receptors that are especially important for helping a person maintain balance are
located in the
a. Gyrus cinguli d. ossicles.
b. Inner ear e. ligaments.
c. Tendons
11. After constant exposure to a stimulus, our nerve cells fire less frequently. This can be
explained by which of the following?
a. Weber’s Law d. absolute threshold sensitivity.
b. Sensory adaptation e. the phi phenomenon.
c. Signal detection
12. Keisha recently lost the sight in her left eye. In which of the following situations would
Keisha NOT be able to judge depth?
a. Looking down a railroad track as it disappears into the distance
b. Looking out a train window passing scenery
c. Throwing a ball to a friend
d. Watching two people approach from different distances
e. Watching a crowd of children at play in a school yard
13. The ability of the human visual system to barely detect a candle flame at a distance of
about 30 miles on a clear, dark night is an example of
a. An absolute threshold d. Weber’s law.
b. A difference threshold e. Fechner’s law.
c. A just noticeable difference
14. The gate control theory attempts to explain how
a. The brain analyzes patterns by analyzing changes in brightness over broad areas
31
b. Filtering of excess information results in reduced response to constant
stimulation
c. The nervous system blocks or allows pain signals to pass to the brain
d. Differing regions of the tongue are more sensitive to different tastes
e. Chemicals released by one animal can shape the behavior of a second animal of
the same species
15. The semicircular canals, located in the inner ear, provide important information about
the
a. Apparent distance of sound
b. Air pressure of the surrounding areas
c. Body’s posture and head position
d. Frequency of sound waves
e. Amplitude of sound waves
Unit 4: Consciousness
146. Conscious
148. Depressants
149. Hallucinogens
150. Hypnosis
152. Insomnia
153. Narcolepsy
32
154. Night terrors
156. Nonconscious
157. Opiates
158. Preconscious
162. Stimulants
163. Subconscious/unconscious
166. Withdrawal
33
Released Test Questions
1. The painful experience associated with termination of the use of an addictive substance
is known as
a. Discontinuance d. forced independence.
b. Tolerance e. transduction.
c. Withdrawal
2. Sigmund Freud believed that dream analysis was a useful device for
a. Decreasing repression
b. Sublimating the id
c. Strengthening the superego
d. Displacing instinctual forces
e. Gaining insight into unconscious motives
3. In terms of the effect on the central nervous system, alcohol is most accurately
classified as which of the following types of drug?
a. Depressant d. stimulant.
b. Narcotic e. hallucinogen.
c. Psychoactive
4. Hypnosis has been found useful in the treatment of
a. Pain d. paranoia.
b. Autism e. schizophrenia.
c. Dementia
5. A student participates in a month-long sleep study designed to examine free-running
circadian rhythms. If all time cues are removed, the student’s sleep-wake cycle is likely
to
a. Average about 25 hours
b. Average about 12 hours
c. Average whatever it had averaged when the student began the study
d. Become even more dependent than usual on the student’s activity level
e. Become extremely variable
6. A central nervous system depressant that produces a false feeling of well-being and
efficiency and results in slower reaction time to stimulation is
a. Cocaine d. alcohol.
b. Marihuana e. nicotine.
34
c. Dopamine
7. The psychological effects of alcohol are powerfully influenced by the user’s
a. Expectations
b. Success in developing a social network
c. Agility
d. Intelligence quotient (IQ)
e. Brain dopamine level
8. Which of the following is characterized by a periodic appearance of sleep spindles?
a. Stage 2 sleep
b. Stage 3 sleep
c. Stage 4 sleep
d. REM sleep
e. Night terrors
Unit 5: Learning
170. Chaining
174. Discrimination
175. Extinction
35
176. Fixed interval schedule
178. Generalization
185. Modeling
36
190. Positive punishment
194. Shaping
37
Released Test Questions: 2004 Exam
1. A two-year-old child is frightened by a small dog. A few weeks later the same child sees
a cat and becomes frightened. The child’s reaction is most likely an example of which of
the following?
a. Stimulus discrimination d. sensory preconditioning.
b. Second-order conditioning e. spontaneous recovery.
c. Stimulus generalization
2. Which is a definition of discrimination that most directly applies to classical
conditioning?
a. Prototype matching to organize information into categories
b. Maintaining information in memory thorough repetition
c. Differential treatment, usually negative, based on group membership
d. Recognizing an object as distinct from its surrounding
e. Learning to respond differently to similar stimuli
3. A monkey is conditioned to flinch at the sound of a bell that was previously paired with
a puff of air to the monkey’s cheek. Which of he following explanations would be
consistent with a cognitive interpretation of this conditioning?
a. The animal cannot control its tendency to flinch because the response of
flinching is simply a reflex to the bell
b. The strength of the flinch response is a function of the time interval between the
onset of the bell and the air puff
c. The monkey interprets the bell as a signal that the air puff will follow
d. The bell is merely a substitute stimulus for the air puff
e. Monkeys are intelligent and know that they should flinch when they hear tones
that are paired with stimuli that elicit reflexes
4. Research indicates that many animals are more likely to associate sickness with a taste
they experienced in conjunction with the illness than with a tone or light. This finding
supports which of the following claims?
a. The tone or light must not have been appropriately paired with the onset of
illness
b. Illness is not necessarily punishing to subject
c. Animals may be biologically prepared to learn some things over other things
d. Extrinsic reinforcers may be more effective than intrinsic reinforcers
38
e. Positive reinforcers are more effective than punishers
5. Rats in an experiment learned to associate sweetened water with a drug that causes
immune suppression. Later, the sweetened water alone produced the immune
suppression. This outcome is an example of which of the following?
a. Learned helplessness d. classical conditioning.
b. Systematic desensitization e. biofeedback.
c. Operant conditioning
6. John Garcia showed that when rats ingested a novel substance before becoming
nauseated from radiation or drugs, they acquired a
a. Conditioned taste preference for the substance
b. Generalized taste preference for similar substances
c. Conditioned taste aversion for the substance
d. Conditioned taste aversion for any novel substance
e. Conditioned taste preference for any novel substance
7. After seeing her parents give her brother a dollar for cleaning his room, Sarah begins
cleaning for own room. According to social-learning theorists, Sarah’s behavior is an
example of which of the following?
a. Classical conditioning d. discrimination training.
b. Spontaneous recovery e. observational learning.
c. Stimulus generalization
204. Chunking
208. Exemplar
210. H.M.
40
212. Informational processing model of memory
221. Prototype
222. Recall
224. Recognition
41
226. Retrieval
235. Algorithm
42
240. Functional fixedness
241. Heuristic
244. Morphemes
246. Overgeneralization
247. Phonemes
249. Syntax
43
Memory Released Test Questions: 2004 Exam
7. Two groups of participants in a study are presented a list of 20 words. The first group
is told to count the number of capital letters in the words and the second group is told
to think of the definition of each word. When both groups are asked to recall the words
lists, which of the following is most likely to occur?
a. Each group will recall the same number of words.
b. The first group will recall more words than the second group.
c. The first group will rehearse the words, but the second group will not.
d. The second group will recall more words than the first group.
e. Both groups will recall all of the words.
8. An example of episodic memory is the memory of
a. One’s high school graduation
b. The capital of a state
c. What the musical note C sounds like
d. How to type
e. A mood that is triggered by the experience of a particular scent
9. The process of remembering several pieces of information by mentally associating an
image of each with a different location is a mnemonic device known as
a. The key-word method
b. The method of loci
c. The peg word system
d. The link method
e. chunking
47
258. General Adaptation Syndrome
259. Incentives
260. Instinct
48
Released Test Questions: 2004 Exam
1. Rudolph spends hours painting in his studio, even though he sells few pictures. Which
of the following explains Rudolph’s creative productivity?
a. Functional fixedness
b. Inductive reasoning
c. Intrinsic motivation
d. Incubation
e. Heuristics
2. Individuals who accept personal responsibility for their life experiences may be
characterized as having
a. Unrealistic expectations
b. Delusions of grandeur
c. An internal locus of control
d. A pessimistic view of reality
e. An introverted personality
3. Which of the following is most likely to characterize the behavior of students who have
high achievement motivation and are intrinsically motivated to play a musical
instrument?
a. If promised a reward for practicing a difficult piece of music selected by the
teacher, they will practice more than if they selected the piece by themselves
b. If permitted to choose their own pieces of music, they will select very difficult
ones that are beyond their present ability to play
c. If permitted to choose their own pieces of music, they will select easy pieces that
they can master in one session
d. If permitted to choose their own pieces of music, they will select moderately
difficult pieces that they can master if they practice them conscientiously
e. They will enjoy practicing the piano more if their parents promise them a
reward for mastering each piano piece
49
4. In Abraham Maslow’s hierarchy of needs, living up to one’s potential and striving for
personal fulfillment are referred to as
a. Biological needs d. belonging needs.
b. Aesthetic needs e. self-actualization needs.
c. Physiological needs
5. Which of the following terms is used in hunger and weight-control research to denote
the concept that each person has a body-fat level that remains fixed and resistant to
change?
a. Hyperphagia
b. Hypophagia
c. Glucagon theory
d. Set point
e. Metabolic conversion
6. The two components of Schachter and Singer’s theory of emotion are
a. Love and anger
b. Instinct and motivation
c. Instinct and brain activation
d. Physiological arousal and cognitive assessment
e. Physiological arousal and behavior
7. Greg stays up all night during finals week studying for exams. As the week progresses,
his muscles tighten and he develops a stiff neck. By the last day of finals, he is taking
more frequent breaks, leaning back in the desk chair, and staring off into space. He
arrives for the last test with a sore throat and headache.
Which of the following best describes Greg’s response to stress?
a. General adaptation syndrome
b. Object-relations theory
c. Opponent-process theory
d. Two-factor theory
e. Type B behavior pattern
8. A teacher taught her students to take turns by giving them stars to trade for snacks at
the end of the day. This technique is called
a. Systematic desensitization
b. Token economy
c. Classical conditioning
d. Rational-emotive therapy
e. Free association
13. According to Abraham Maslow’s hierarchy of needs, which of the following needs must
first be met before belongingness and love become important?
a. Safety only
b. Safety and physiological needs
c. Esteem and self-actualization needs
d. Safety, esteem, and physiological needs
e. Esteem, self-actualization, and physiological needs
14. Which of the following lends the most support to the view that early explorers
undertook their explorations to gain wealth and fame?
a. Arousal theorists d. incentive theorists
b. Drive-reduction theorists e. sociobiological theorists
c. Humanistic theorists
15. Which of the following is the phase of Hans Selye’s general adaptation syndrome
wherein individuals are most vulnerable to illness, collapse, and even death?
a. Appraisal d. denial
b. Alarm e. resistance
c. Exhaustion
16. According to David McClelland, which of the following is true of individuals who have a
strong need for achievement?
a. They seek out tasks that are moderately difficult
b. They are not likely to persist on any task
c. They seek out tasks that are easy
d. They seek out tasks that are nearly impossible
e. They are less likely to pursue success in their occupations
17. The Schachter-Singer theory of emotion includes a component not discussed in the
James-Lange theory. What is this component?
a. Stimulus situation d. cognitive labeling
b. Arousal e. subjective emotion
c. Facial feedback
51
18. A researcher gave identical puzzles to five-year-old children in two groups, group A and
group B. The children in group A were promised candy if they completed the puzzles
quickly and well. They children in group B were not promised a reward but were
encouraged to do the best that they could. If the researcher argues that
overjustification occurred, which of the following most likely happened during the
experiment?
a. Group A worked harder and better than group B
b. Both groups worked equally hard and well
c. Group B felt angry at not receiving the promise of a reward and therefore did not
work at all
d. Group A did not work as hard or as well as group B and would be less likely to
spontaneously work on similar puzzles in the future
e. Group B worked equally well as group A but would quickly argue that they
would have done better if offered a rewards.
267. Accommodation
269. Assimilation
52
274. Concrete operational stage
275. Conservation
278. Egocentric
53
288. Jean Piaget
292. Metacognition
300. Schema
54
302. Sensorimotor stage
303. Teratogens
1. Which of the following theoretical frameworks would argue most strongly that a
healthy child will choose what is good for his or her own growth?
a. Social learning theory d. Humanistic
b. Psychoanalytic e. Psychodynamic
c. Behavioral
2. A researcher asks elementary, junior high, senior high, and college students to define
the term “cheating,” and analyze differences in their definitions across age groups. This
is an example of which type of study?
a. Longitudinal d. Case study
b. Sequential e. Observational
c. Cross-sectional
3. Keisha was able to roll over at two months, crawl at five months, and walk at ten
months. This sequence of development is most likely due to
a. Egocentrism d. Attachment
b. Social context e. Assimilation
c. Maturation
4. A researcher dabs color on a 16-month-old child’s face and places the child in front of a
mirror. Which of the following developmental milestones has been reached if the child
realizes that there is something wrong with its face?
a. Visual discrimination d. Identification of the gender of the
image
b. Recognition of human form e Perception of the image as a playmate
c. Recognition of self
5. Which of the following reflects the inborn and stable rudiments of personality, such as
excitability?
55
a. Imprinting d. Temperament
b. Social referencing e. Accommodation
c. Individuation
6. A 14-moth-old toddler is placed in an unfamiliar situation with the child’s mother, who
then leaves the room for a time. When the mother returns, the child squirms and tries
to get away from the mother when picked up, but also seems distressed when placed
back on the floor. Mary Ainsworth would consider this evidence of which of the
following?
a. Hyperactivity d. Disorganized Behavior
b. Narcissistic personality type e. Avoidance
c. A resistant or ambivalent attachment style
7. Which Piagetian stage of cognitive development is characterized by mastery of
conservation tasks?
a. Sensorimotor d. Formal Operation
b. Preoperational e. Tertiary circular reactions
c. Concrete operations
8. According to Lawrence Kohlberg, children at the first level of moral reasoning make
judgments about right and wrong on the basis of
a. Cultural norms d. The likelihood of punishment
b. Social conventions e. The approval of siblings or same-sex
peers
c. Religious beliefs
9. Gender typing is best described as
a. The realization that gender is stable, regardless of superficial behaviors such
as dress
b. A preferential interest of a female child in the attention of the mother
c. A child showing ambivalence toward the mother and father
d. Adolescents integrating social, sexual, and intellectual, and ethical aspects
into self-identity
e. The acquisition of the roles associated with distinctions between male and
female
10. In “Baby X” studies, researchers alternately dressed an infant in typical boys’ or girls’
clothing. Adults who were asked to play with the infant treated the baby in boys’
clothing in a generally rougher manner than when asked to play with the same infant
dressed in girls’ clothing. This behavior is the result of
a. Stereotyping d. Neglect
b. Androgyny e. Framing
c. Indulgence
11. A child believes that all birds can fly, yet comes to realize that a penguin is a bird.
Which of the following terms was used by Jean Piaget to refer to this developmental
process of changing one’s ways of thinking in order to incorporate new information?
56
a. Accommodation d. Assimilation
b. Egocentrism e. Concrete operational thought
c. Formal operational thought
12. Maturation differs from learning in that developmental changes due to maturation
a. Are relatively independent of environmental factors
b. Follow no predictable pattern
c. Can take place only in an enriched environment
d. Cease after early childhood
e. Require practice
13. A mother gives her child an orange for the first time. The child rolls it across the table
and says, “ball.” Which of the following Piagetian processes would account for his
behavior?
a. Seriation d. Assimilation
b. Accommodation e. Conservation
c. Reversal
14. According to Erik Erikson’s theory of development, the key concern of adulthood is
a. Establishing a career d. Sharing intimacy with another
b. Leaving the parent’s world e. Building an identity
c. Raising children
Unit 9: Personality
306. Archetypes
311. Compensation/overcompensation
57
312. Defense Mechanisms
313. Denial
314. Displacement
315. Ego
316. Extrovert
321. Heritability
322. Id
324. Introvert
58
326. MMPTI-2
328. Projection
331. Rationalization
334. Regression
335. Repression
336. Self-efficacy
337. Sublimation
338. Superego
59
340. Unconditional positive regard
1. Individuals who accept personal responsibility for their life experiences may be
characterized as having:
a. Unrealistic expectations
b. Delusions of grandeur
c. An internal locus of control
d. A pessimistic view of reality
e. An introverted personality
2. The technique of assessing personality by asking a person to make up a story based on a
picture presented by the researcher is an example of which of the following types of
tests?
a. Trait/type
b. Projective
c. Forced choice
d. Simulation
e. Personality inventory
3. The humanistic perspective in psychology suggests that
a. Most behavior can be explained by operant conditioning principles
b. All humans have peak experiences
c. All humans are good by nature
d. Humans are doomed to a life of neurosis and suffering
e. Self-actualization can happen only through therapy
4. Which of the following assessment tools is LEAST likely to be used by a
psychoanalytically oriented psychologist?
a. Sentence completion
b. Minnesota Multiphasic Personality Inventory
60
c. Draw-A-Person Test
d. Thematic Apperception Test
e. Rorschach Test
5. Which perspective views the human condition as a mixture of unconscious desires and
conflicts?
a. Humanistic
b. Trait
c. Psychoanalytic
d. Behavioral
e. Cognitive
6. Which perspective was used by Raymond Cattell as a basis for his personality measure
known as the 16PF test?
a. Humanistic
b. Trait
c. Psychoanalytic
d. Behavioral
e. Cognitive
61
10. The five-factor theory of personality (The Big Five) does NOT include which of the
following characteristics?
a. Intelligence
b. Conscientiousness
c. Extraversion
d. Agreeableness
e. Neuroticism/stability
11. Which of the following did Alfred Adler suggest is the primary driving force in the
development of personality?
a. Resolving the Electra complex
b. Overcoming the inferiority complex
c. Extinguishing reinforcement history
d. Analyzing the collective unconscious
e. Repressing the libido
12. Most professionals from the United States rate themselves better in performance and
competence than their average peer. This is an example of
a. Self-actualization
b. Objective self-awareness
c. Enlightened self-schemas
d. Self-serving bias
e. The fundamental attribution error
Unit 10: Intelligence
347. Heritability
62
348. Negatively skewed
351. Reliability
358. Validity
63
Released Test Questions: 2007 Exam
2. In a normal distribution, approximately what percent of the scores occur within one standard
deviation above and below the mean?
a. 5%
b. 16%
c. 33%
d. 68%
e. 97%
3. Jamal receives a Full Scale IQ score of 125 on the Stanford-Binet Intelligence Scales. This score
indicates that he
a. Scored correctly on 125 items on the test
b. Scored exactly at the mode of the test
c. Scored exactly at the median of the test
d. Scored more than one standard deviation above the mean of the test
e. Has 125 units of intelligence as operationalized by the test
4. When a teacher compares the performance of her students on the even-and-odd numbered
questions in a multiple-choice test, she is determining
a. Equivalent-form reliability
b. Split-half reliability
64
c. Face validity
d. Concurrent validity
e. Construct validity
6. Which of the following is generally true of participants in Lewis Terman’s longitudinal study of
intellectually gifted children?
a. Their IQ dropped with age
b. Their IQ increased with age
c. They exhibited a higher-than-average incidence of mental illness
d. They led happy and fulfilling lives
e. They came from larger-than-average families
7. The correlations between the IQ scores of identical twins reared apart are lower than those of
identical twins reared together. This difference is best explained by which of the following?
a. Heredity plays an important role in determining IQ
b. Environment plays an important role in determining IQ
c. Heredity plays no role in determining IQ
d. Environment plays no role in determining IQ
e. Heredity and environment play an equal role in determining IQ
10. Alfred Binet’s most important contribution to psychology was in the area of
a. Intelligence testing
b. Visual perception
c. Psychopathology
d. Comparative psychology
65
e. Classical conditioning
11. Which of the following is evidence of the reliability of a new intelligence test?
a. A correlation of +.90 exists between scores on the new test and scores on the Wechsler
Intelligence Scale for Children
b. The test predicts students’ ability to succeed in college
c. The correlation between scores for identical twins taking the test is +.90
d. Baseline data for test norming are obtained from a diverse sample of several thousand
participants
e. The correlation between scores of participants who take two forms of the test +.90
362. Agoraphobia
66
366. Antisocial personality disorder
374. Counterconditioning
376. Delusion
67
380. Diathesis-stress model
388. Flooding
393. Hallucination
68
394. Histrionic personality disorder
395. Hypochondriasis
405. Phobia
407. Psychiatrist
69
408. Psychoanalysis
409. Psychosurgery
411. Schizophrenia
418. Transference
70
1. The basic purpose of the DSM-IV-TR is to
a. provide a set of diagnostic categories for classifying psychological disorders.
b. provide a clear distinction between neurosis and psychosis.
c. describe the psychoanalytic approach to psychological disorders.
d. describe internal personality factors that are involved in psychological disorders.
e. identify childhood experiences that contribute to psychological disorders.
2. An individual survives a period of captivity and exhibits behaviors that include anxiety,
inability to concentrate, depression, edginess, and the reexperience of stressful events. These
symptoms illustrate which of the following disorders?
a. Generalized anxiety d. Histrionic
b. Major depression e. Posttraumatic Stress
c. Hypochondriasis
3. Contemporary definitions of abnormality might include all of the following criteria EXCEPT
a. maladaptiveness. d. gender.
b. personal distress or discomfort. e. statistical prevalence.
c. deviance from cultural norms.
4. Which of the following forms of therapy most likely involves a confrontational atmosphere
between the therapist and the client?
a. Rational-emotive therapy d. Person-centered therapy
b. Psychoanalysis e. Systematic desensitization
c. Aversive conditioning
5. The overall purpose of psychoanalytic therapy is to
a. change the behavior patterns of the client in therapy.
b. help the client in therapy reach his or her full potential.
71
c. alter the thought processes of the client in therapy.
d. help the client in therapy gain new insight into himself or herself.
e. help the client in therapy get rid of irrational thoughts.
6. An individual who sees and feels imaginary spiders crawling on his arms and legs is
experiencing
a. a fixation. c. an illusion. e. a phobia.
b. a hallucination. d. an eidetic image.
7. According to the psychoanalytic view, depression is caused by
a. a neurotransmitter imbalance.
b. self-defeating thoughts.
c. prolonged exposure to stressor over which the individual has limited control.
d. unresolved experiences of loss from childhood.
e. alcohol or other substance abuse.
8. One perspective in clinical psychology in clinical psychology proposes that adaptive and
abnormal behavior can be developed through similar processes. Which of the following
terms best characterizes this approach to abnormal behavior?
a. Biological c. Behavioral e. Cognitive
b. Psychodynamic d. Humanistic
9. Which of the following treatments is most frequently used to eliminate specific phobias?
a. Antidepressant drugs d. Psychoanalysis
b. Systematic desensitization e. Aversion therapy
c. Implosion therapy
10. A college student experiences a loss of sensation in her right arm before exams. Doctors can
find physiological basis for her condition. This student is most likely experiencing which of
the following kinds of disorders?
a. Somatoform c. Anxiety e. Personality
b. Dissociative d. Mood
11. The psychologists who first developed encounter groups and sensitivity-training groups
based their work on which of the following approaches to therapy?
a. Psychodynamic c. Behavioral e. Biological
b. Humanistic d. Cognitive
12. Prozac functions as an antidepressant medication because it
a. enhances production of acetylcholine.
b. blocks the reuptake of the neurotransmitters serotonin.
c. causes selective memory loss for depression-producing events.
d. produces a steady, mild state of euphoria.
e. inhibits front lobe activity related to depression.
13. A teacher taught her students to take turns by giving them stars to trade for snacks at the end
of the day. This technique is called
a. systematic desensitization. d. rational-emotive therapy.
b. token economy. e. free association.
c. classical conditioning.
14. Aaron Beck suggested that negative beliefs cause depression. To help change these negative
beliefs, Beck used which of the following therapies
a. Cognitive d. Psychoanalytic
b. Psychopharmacological e. Social-learning
c. Rational-emotive
15. Which of the following is most descriptive of antisocial personality disorder?
72
a. A pattern of limited social interaction and reluctance to enter into relationships
b. A pattern of extreme dependence on other people and acute anxiety at being left alone
c. A pattern of bizarre or unstable behavior characterized by dramatic mood shifts
d. An inability to feel empathy for others and a lack of remorse for actions that harm
others
e. An exaggerated sense of self-importance
16. Rhoda sometimes experiences periods of major depression. At other times, she is extremely
talkative and active, appears to be in a euphoric mood, goes days without sleeping, and
reports that her thoughts are racing. The most likely diagnosis of Rhoda’s condition is
a. bipolar disorder. d. major depressive disorder.
b. dissociative identity disorder. e. schizophrenia.
c. delusional disorder.
17. Dan always return after leaving home to see if he locked the door. He always locks the door,
but he thinks about whether or not he locked until he has checked. Dan’s behavior is an
example of
a. a specific phobia. d. conversion disorder.
b. avoidant personality disorder. e. obsessive compulsive disorder
c. hypochondriasis.
18. The Diagnostic and Statistical Manual of Mental Disorder (DSM-IV-TR) includes which of
the following?
a. Detailed case histories and verbatim patient interviews for each type of mental
disorder
b. Table of biological abnormalities responsible for each type of mental disorder
c. Discussions of the pros and cons of various treatment approaches for each type of
mental disorder
d. Classification and diagnosis of each type of mental disorder
e. Discussions of insurance coverage for each type of mental disorder
19. Aversion therapy to eliminate undesirable behaviors is most likely to be used by which of the
following types of therapists?
a. Behavioral c. Cognitive e. Humanistic
b. Psychoanalytic d. Reality
20. A therapist who uses systematic desensitization to help a child overcome a white rabbit
phobia will probably begin treatment by
a. exploring the child’s early ego formation.
b. having the child imagine being in a room full of rabbits.
c. having the child list a hierarchy of white, fuzzy objects.
d. analyzing the child’s recent dreams for evidence of rabbit themes.
e. exposing the child to several tame, docile rabbits.
21. A person leaves her current home, moves to a new location, takes on a new identity, and has
little or no memory of her former life. According to the Diagnostic and Statistical Manual of
Mental Disorders (DSM-IV-TR), which of the following diagnoses is most appropriate?
a. Conversion disorder d. Dissociative Fugue
b. Panic disorder e. Schizophrenic episode
c. Bipolar disorder
73
22. A therapist who helps a client replace irrational thoughts with alternative patterns of thinking
is using which of the following techniques?
a. Cognitive restructuring d. Extinction
b. Cognitive dissonance e. Aversive conditioning
c. Systematic desensitization
23. In the treatment of patients, which of the following psychologists made no attempt to offer
formal diagnosis, advice, or interpretation of patients’ unconscious motives?
a. Carl Rogers c. Albert Ellis e. Carl Jung
b. Aaron Beck d. Sigmund Freud
24. Kevin laughs when others are injured by his thoughtless pranks and shows little regard for
the feelings or well-being of others. He has been in trouble with the police and other
authority figures a number of times. Which of the following disorders would best describe
Kevin’s behavior?
a. Dissociative Identity Disorder d. Obsessive compulsive disorder
b. Antisocial personality disorder e. Dependent personality disorder
c. Narcissistic personality disorder
25. Client: “I’ve been feeling totally worthless, like I never do anything right.”
Therapist: “I guess you feel that way a lot, don’t you? That you’re no good. It’s hard to have
feelings like that.” Which of the following types of therapy would be likely to involve the
exchange above?
a. Rational-emotive therapy d. Systematic desensitization
b. Gestalt therapy e. Person-centered therapy
c. Psychoanalysis
26. Which of the following psychoactive drugs works by inhibiting the reuptake or serotonin?
a. Xanax c. Lithium carbonate e. Prozac
b. Valium d. Clozapine
27. Although it can produce undesirable side effects, electroconvulsive therapy (ECT) has been
shown to be most effective in the treatment of
a. depression. d. psychogenic amnesia.
b. schizophrenia. e. multiple personality.
c. panic disorder.
28. Which of the following is true of persons with schizophrenia?
a. They have multiple personalities.
b. They are typically cured if they take appropriate medication.
c. They are often unable to understand simple statements.
d. They typically experience onset late in adolescence or early adulthood.
e. They typically have a history of being maltreated as children.
29. Dr. Williams is a therapist who believes that his client’s depression is caused by internalized
anger and other intra-psychic conflicts. Of which psychological perspective is Dr. Williams
most likely a proponent?
a. Biopsychological c. Humanistic e.
Psychoanalytic
b. Evolutionary d. Behavioral
30. Which of the following best represents a humanistic explanation for anxiety disorder?
a. Lower-than-normal levels of inhibitory neurotransmitters like GABA result in
higher levels of anxiety.
b. Irrational and maladaptive thoughts processes lead to increased levels of anxiety.
c. Anxiety is either classically or operantly conditioned.
d. Anxiety results from internal unconscious conflict.
74
e. Anxiety results when an individual experiences a gap between his ideal self and
his real self.
31. Janet believes her brother Ralph is depressed because he has a distorted view of reality: he
has unrealistically negative thoughts. Which of the following theories comes closest to
describing Janet’s orientation to abnormal behavior?
a. Biological c. Behavioral e. Cognitive
b. Psychodynamic d. Systems
425. Compliance
75
426. Foot-in-the-door strategy
436. Stereotype
437. Prejudice
438. Discrimination
439. Ethnocentrism
76
440. In-group bias
446. Conformity
453. Deindividuation
77
454. Zimbardo’s prison experiment
78
4. Which of the following explains the behavior of normally law-abiding people who act
destructively when they are part of a crowd?
a. Group polarization d. Entrapment
b. The mere exposure effect e. Fundamental attribution error
c. Deindividuation
5. A club president discovers that contributions of club members drop when the total
contribution of all members is published rather than the contributions of individuals.
This drop can be explained by the phenomenon of
a. group polarization d. social facilitation
b. learned helplessness e. socialization
c. social loafing
6. Two friends attribute their high math scores to their high level of effort and ability in
math and their low Spanish scores to their teacher’s subjective grading and favoritism. In
this situation these students are exhibiting
a. the actor-observer effect. d. equity.
b. the hindsight bias. e. the self-serving bias.
c. scapegoating.
7. All of the following are conditions that may lead to conflict within organizations
EXCEPT
a. scarce resources. d. insufficient communication.
b. jurisdictional ambiguity. e. superordinate goals.
c. inequities in status.
8. According to the theory of fundamental attribution error, when explaining the failure of
others we usually underestimate the significance of
a. situational factors. d. support systems.
b. dispositional factors. e. inherited traits.
c. motivational factors.
9. Social facilitation theory focuses on situations in which the presence of others causes an
individual’s performance to
a. remain unchanged. d. spontaneously recover.
b. decline. e. become fixated.
c. improve.
10. A group of students perceived a stationary point of light in a dark room as a moving
object. When asked later how far the light had moved, their responses increased in
similarity as they heard each others’ estimates. This outcome was a demonstration of
a. obedience. d. visual masking.
b. social facilitation. e. motion parallax.
c. conformity.
11. Tension from an inconsistency between an individual’s attitudes and behavior is
a. cognitive dissonance. d. information propinquity.
b. altruism. e. constancy.
c. deindividuation.
12. The common tendency to assume that the beliefs, values, attitudes, or actions of one’s
own group are superior to those of other groups is called
a. deindividuation. c. observer bias. e. reactance.
b. groupthink. d. ethnocentrism.
79
13. When members of a group who tend to agree on an issue become more extreme in their
opinion after the issue is discussed, which of the following has taken place?
a. deindividuation d. group norming
b. depersonalization e. group polarization
c. groupthink
14. The frustration-aggression hypothesis views aggression as
a. unconscious. c. innate. e. cognitive.
b. physiological. d. reactive.
15. When Stanley Milgram asked psychiatrist to predict how far participants would go in
administering shock in his original demonstration on obedience to authority, the
psychiatrists
a. made accurate predictions.
b. significantly overestimated the level.
c. significantly underestimated the level.
d. accurately predicted male responses but underestimated female responses.
e. declined to play a role in the research based on their ethical objections.
16. Jessie and a friend are driving in rush-hour highway traffic when a car suddenly cuts in
front of them in order to reach an unmarked exit ramp. Jessie’s passenger points out that
the circumstances may have required the other driver to do so, while Jessie loudly
criticizes the personal qualities of the other driver. Jessie’s response is best explained by
a. cognitive dissonance theory. d. the mere exposure effect.
b. the fundamental attribution error. e. social inhibition theory.
c. a self-fulfilling prophecy.
80
General Info
• 70 minutes to do 100 multiple choice questions (MCT)
• 50 minutes to complete two essays called Free Response Questions (FRQ)
• MCT counts for 2/3rds of your score or 100 points
• FRQ’s count for 1/3 of the score or 50 points (25 points each)
MCT
• The questions are broken down into 3rds. Roughly the first third are easy; the second third
are more difficult and the last third are difficult. That is why you may feel discouraged as
you progress on the test, (not to worry).
• Use the process of elimination approach and guess if you can knock out enough (at least 1)
answer(s)
• Since you know that the first 1/3 is easy you want to do your best there
• Take your time to get them right but don’t spend too much time on it
81
• For the last 1/3 read the question. If you think you know it, go for it and answer. When and
if you are clueless mark it and return to it when/if you have time.
Scoring
1. Each essay is worth 16 2/3rds or 33% of the final AP score
2. Each essay has a specified number of pieces of information required usually between 8 and
12
3. Example of scoring if an essay is worth 10 points you would need to multiply the number
correct by 2.5 in order to get the 25 points total possible. If you scored a 7 on that essay (7
X 2.5 = 17.5) out of 25.
82
MCQ’s Strategies
1. If you encounter a question in which you have no idea of the answer and you cannot at least
eliminate one choice, leave it blank. There are 0.25 points deducted for every wrong answer
and 0 point deducted for every blank answer.
2. When two out of four choices are opposites, pick one of those two as the best guess.
5. In questions asking for the most or the least, pick the answer next to the most or the least. (Ex.
5, 8, 9, 15, 30)
8. If two out of four choices are almost identical, pick the longer of the two.
9. If a question asks for a plural or a singular answer, make sure that you pick a plural or singular
answer.
10. When limiting words are used (all, never, always, must, etc.) “False” is usually the better
answer.
11. When general terms are used (most, some, usually, could, might, etc. ), “true” is usually the
better answer.
12. Identify those questions you’re not sure of with a mark. Review these on a second pass.
13. Reread directions before turning in an exam. Use the entire time given to double-check.
14. First impressions (initial guess) are often best. If an answer comes to you from out of the blue,
it’s probably your right brain at work. Don’t fight this intuition unless you’re sure it’s wrong.
83
FRQ Tips
1. Read both questions before you begin writing. Rubric them, target the total points
that you will need to make.
• Number the points right on the green sheet
• Make marginal notes about terms/research to use
• Refer back to the pink booklet you write on. In a box with question #’s ETS may give
you number of rubric points for each essay.
• Summarize and break down the questions in your own words.
4. Time
• Try to budget 20 minutes per question the last 10 minutes you can use as necessary.
5. Give the readers only what they are looking for and nothing more.
6. Don’t cross out sections, you may have been right. You may cross out single words but
never whole lines or sections. The readers have to give you a point even if you stumble on
it.
7. If you give research examples do it briefly. You get NO extra credit for describing
research in detail. Readers are looking for certain terms and concepts. Research examples
are only good to indirectly “show you know”.
8. Try not to overwrite on one question. Budget your time equally between both questions.
Try to stick to the 20-minute rule.
9. Use psych terms – Define the psych terms – Underline the psych terms.
84
Past AP Free Response Questions – Content Areas
85
Advanced Placement
Program
AP® Psychology
Practice Exam
The questions contained in this AP® Psychology Practice Exam are written to the content specifications of
AP Exams for this subject. Taking this practice exam should provide students with an idea of their general
areas of strengths and weaknesses in preparing for the actual AP Exam. Because this AP Psychology
Practice Exam has never been administered as an operational AP Exam, statistical data are not available
for calculating potential raw scores or conversions into AP grades.
This AP Psychology Practice Exam is provided by the College Board for AP Exam preparation. Teachers
are permitted to download the materials and make copies to use with their students in a classroom setting
only. To maintain the security of this exam, teachers should collect all materials after their administration
and keep them in a secure location. Teachers may not redistribute the files electronically for any reason.
© 2008 The College Board. All rights reserved. College Board, Advanced Placement Program, AP, AP Central,
SAT, and the acorn logo are registered trademarks of the College Board. PSAT/NMSQT is a registered trade-
mark of the College Board and National Merit Scholarship Corporation. All other products and services may
be trademarks of their respective owners. Visit the College Board on the Web: www.collegeboard.com.
86
Section I
Multiple-Choice Questions
87
-1-
PSYCHOLOGY
SECTION I
Time—1 hour and 10 minutes
100 Questions
Directions: Each of the questions or incomplete statements below is followed by five suggested answers or
completions. Select the one that is best in each case and place the letter of your choice in the corresponding box on
the student answer sheet.
1. When asked which of two countries has a larger 4. Which psychologist reported that infant
population, participants are likely to judge the attachment to another goes beyond the
country that is more familiar to them as being satisfaction of the need for nourishment?
more populous. Which of the following best
(A) Albert Bandura
explains this finding?
(B) Jean Piaget
(A) A means-end analysis (C) Harry Harlow
(B) The representativeness heuristic (D) Erik Erikson
(C) The availability heuristic (E) Konrad Lorenz
(D) Algorithms
(E) Inductive reasoning 5. Edward L. Thorndike argued that responses that
lead to satisfying outcomes are more likely to be
2. An individual with damage to Wernicke’s area repeated, and that responses followed by
is most likely to have difficulty unpleasant outcomes are less likely to be repeated.
This became known as the law of
(A) identifying an object held in the hand but
not seen (A) reinforcement
(B) planning what to wear to a party (B) associations
(C) remembering the name of a person in a (C) punishment
photograph (D) effect
(D) comprehending a spoken request for (E) outcomes
information
(E) distinguishing between red and green 6. When trying to solve a problem, Bret uses
a logical, step-by-step formula called
3. The psychoanalytic concept of repression
(A) a heuristric
suggests a difficulty in the functioning of
(B) incubation
which aspect of memory?
(C) insight
(A) Encoding (D) priming
(B) Short-term memory (E) an algorithm
(C) Procedural memory
(D) Explicit memory
(E) Retrieval
88
GO ON TO THE NEXT PAGE.
-2-
7. It is well established that certain autonomic 10. Researchers find that there is a significant,
responses such as heart rate, perspiration, positive correlation between the number of hours
and respiration change under stress. In view students sleep and their grades. The researchers
of the fact that people generally have stronger would be justified in concluding that
autonomic responses when lying than when
(A) earning good grades causes people to sleep
telling the truth, it follows that the polygraph
more
would be a foolproof approach to lie detection.
(B) sleeping more causes students to perform
Which statement best explains why the polygraph
better in school
is not more widely used in courtrooms and in
(C) students who earn good grades tend to sleep
testing of job applicants?
more than those who do not
(A) Most people can avoid detection when (D) more sleep has a beneficial impact on
they lie. students’ grades
(B) Physiological arousal is much the same (E) sleep deprivation has no impact on school
for several emotions, so the polygraph performance
cannot always reliably distinguish guilt
from other reactions. 11. In phase one of a study, a researcher classically
(C) A significant number of people show conditions a dog to salivate to the ringing of
paradoxical autonomic reactions, a bell. In the second phase, the researcher pairs
responding more strongly when telling a flashing light with the ringing of the bell.
the truth than when lying. After several pairings of the light and the bell,
(D) In controlled studies, the polygraph has the dog will
correctly identified guilty individuals
(A) no longer salivate when the bell is rung
in only a small percentage of cases.
(B) only salivate when the bell is rung
(E) The polygraph has been shown to be
(C) salivate when the light is flashed
reliable only in highly emotional cases,
(D) stop salivating when the light is flashed
such as child abuse and spying.
(E) salivate when the researcher comes into
the room
8. The principles of operant conditioning are best
illustrated by
12. Jason is attending a parade that features the local
(A) exposing a client to anxiety-provoking high school band. Jason’s friend Brent plays the
stimuli trombone in the band. It is difficult for Jason to
(B) replacing a response to a stimulus with an hear Brent play at the parade. Which
alternative response of the following would best allow Jason to hear
(C) deep relaxation techniques Brent’s trombone?
(D) a token economy to reinforce adaptive
(A) Sensory adaptation
behaviors
(B) Selective attention
(E) development of intrinsic motivation
(C) Perceptual constancy
(D) Weber’s law
9. A man is feeling depressed about his inability
(E) Functional fixedness
to support his family after losing his job. The
fact that the patient is currently unemployed
is coded on which axis in the Diagnostic and
Statistical Manual of Mental Disorders
(DSM-IV-TR)?
(A) Axis I
(B) Axis II
(C) Axis III
(D) Axis IV
(E) Axis V
89
GO ON TO THE NEXT PAGE.
-3-
Questions 13-15 refer to the information below. 16. A statistical technique that would allow
a researcher to cluster such traits as being
A psychologist describes the following steps to talkative, social, and adventurous with
a client, while the client is practicing relaxation extroversion is called
techniques, in order to treat the client’s psychological
(A) a case study
disorder:
(B) meta-analysis
(C) statistical significance
1. You are entering a large building.
(D) factor analysis
2. You are pressing a button for an elevator.
(E) z score
3. You are stepping into an elevator.
4. You are watching the doors close
17. Ralph is aware that smoking is harmful to his
after entering the elevator.
health, but he continues to smoke. According to
5. You are traveling five floors on
cognitive dissonance theory, it is most likely that
the elevator.
Ralph will
13. Which of the following categories would (A) gather information on the dangers of smoking
most likely represent the diagnosis associated (B) start smoking more frequently
with this set of behaviors? (C) argue that his friends should stop smoking
(D) focus on the social advantages to smoking
(A) Mood disorder
(E) experience no tension
(B) Anxiety disorder
(C) Dissociative disorder
18. Curare blocks action at acetylcholine synapses
(D) Schizophrenic disorder
and causes paralysis. This drug is an example
(E) Somatoform disorder
of an
14. What type of treatment technique is described (A) antagonist
above? (B) agonist
(C) inhibitory postsynaptic potential (IPSP)
(A) Aversion therapy
(D) excitatory postsynaptic potential (EPSP)
(B) Systematic desensitization
(E) excitatory neurotransmitter
(C) Extinction
(D) Flooding
(E) Punishment
90
GO ON TO THE NEXT PAGE.
-4-
19. Which of the following graphs shows the most desirable test-retest correlation?
(A)
(B)
(C)
(D)
(E)
20. Which of the following is a type of medication 22. Which theoretical perspective in psychology
that has been linked to the side effect of tardive attempts to characterize the way in which humans
dyskinesia? store and process sensory information?
(A) Antidepressants (A) Behavioral
(B) Benzodiazepines (B) Psychodynamic
(C) Neuroleptics (C) Evolutionary
(D) Anxiolytics (D) Cognitive
(E) Tricyclics (E) Sociocultural
21. A researcher surveyed social adjustment in the 23. Gestalt psychology is concerned primarily
same group of 20 people from early childhood with understanding which of the following?
through adulthood. In this example, the group
(A) Learning
of 20 people surveyed was the study’s
(B) Motivation
(A) Sample (C) Development
(B) Population (D) Sensation
(C) Operational definition (E) Perception
(D) Control group
(E) Randomization
91
GO ON TO THE NEXT PAGE.
-5-
24. During the night, Alicia stops breathing 28. Which of the following statements best describes
repeatedly, frequently gasps for air, and snores the role of biological processes in classical
loudly at regular intervals. Alicia is most likely conditioning?
suffering from which of the following conditions?
(A) A biologically-based unconditioned stimulus
(A) Sleep apnea (UCS) must immediately follow a
(B) Narcolepsy conditioned stimulus (CS) for learning
(C) Insomnia to occur.
(D) Night terrors (B) Any novel or familiar stimulus could serve
(E) The REM rebound effect as a CS because the biological mechanisms
underlying learning are very powerful.
25. All summer Thomas hears the sound of the ice- (C) Because all animals share a common cellular
cream truck approaching before his brother Oscar history, the laws of classical conditioning
hears it. Thomas most likely has which of the apply to all species.
following? (D) Certain species are biologically predisposed
to learn particular associations that enhance
(A) A lower absolute threshold for hearing
their survival.
than Oscar
(E) Biological reinforcers foster learning more
(B) A greater difference threshold for hearing
quickly than do environmental reinforcers.
than Oscar
(C) A deficit in a sensory system other than
29. Respondents to surveys and questionnaires often
hearing
report that they are healthier, happier, and less
(D) A greater amount of experience with
prejudiced than would be expected based on the
approaching ice-cream trucks than Oscar
results of other types of research. This finding
(E) A tendency for confabulation
can best be explained by which of the following?
26. A therapist who emphasizes helping clients (A) Sampling bias
to identify and change irrational beliefs that (B) Experimenter bias
underlie feelings of anxiety is using which (C) The social desirability bias
therapeutic technique? (D) The bystander effect
(E) The placebo effect
(A) Client-centered therapy
(B) Rational-emotive behavior therapy
30. Synesthesia is a phenomenon that has been
(C) Aversion therapy
estimated to occur in only a few people in
(D) Psychoanalytic therapy
a million. Because of its rarity, researchers
(E) Social-learning therapy
are likely to choose which research method
to study it?
27. Julio has fragmented thinking and distorted false
beliefs. Which of the following psychological (A) Naturalistic observation
disorders is Julio most likely experiencing? (B) Correlational research
(C) Survey research
(A) Simple phobia
(D) Case study
(B) Somatization disorder
(E) Experimental research
(C) Antisocial personality disorder
(D) Obsessive-compulsive disorder
(E) Schizophrenia
92
GO ON TO THE NEXT PAGE.
-6-
31. Paula lacks self-confidence. She has a difficult 35. In a classic study, a group of rats learned to run
time expressing disagreement with others, and through a maze to obtain food, and another group
she usually lets friends make decisions for her. of rats explored the maze without receiving food.
Others have commented that they do not know Some time later, the researcher compared the two
who the “real” Paula is. With which of the groups of rats to determine if both groups would
following personality disorders might Paula find the food at the end of the maze. According to
be diagnosed? the researcher, the untrained rats found the food at
the end of the maze as quickly as the trained rats
(A) Histrionic
as a result of
(B) Borderline
(C) Dependent (A) latent learning
(D) Passive-aggressive (B) observational learning
(E) Narcissistic (C) avoidance learning
(D) counterconditioning
32. Chuck recalls the day last summer when he fell (E) aversive conditioning
off his bicycle and scraped his knee. This is an
example of 36. During a psychology experiment, a researcher
uses a probe to lesion the ventromedial nucleus
(A) iconic memory
of a rat’s hypothalamus. After the procedure
(B) procedural memory
the rat most likely will
(C) semantic memory
(D) episodic memory (A) become less aggressive
(E) short-term memory (B) become more aggressive
(C) eat more and gain weight
33. Researcher Renee Baillargeon found that (D) stop eating and lose weight
four-month-old infants will look longer at (E) experience a loss of coordination
a ball if it appears to roll through a solid barrier, and muscular control
demonstrating that babies seem to grasp basic
physical laws intuitively. Which of the following 37. When a person is suffering from severe pain,
theories does this finding challenge? the type of drug that will best help alleviate
that pain is
(A) Lawrence Kohlberg’s theory of moral
development (A) an opiate
(B) Lev Vygotsky’s social cognition learning (B) an amphetamine
model (C) a depressant
(C) Jean Piaget’s theory of cognitive (D) a hallucinogen
development (E) a stimulant
(D) Mary Ainsworth’s findings from the Strange
Situation 38. The defense mechanism of projection is best
(E) Erik Erikson’s theory of psychosocial illustrated by which of the following examples?
development
(A) When scolded by his parents, a college
student reverts to childlike behavior to
34. In vision, transduction occurs within the
gain sympathy.
(A) optic nerve (B) A soccer player who does not have much
(B) visual cortex athletic skill constantly criticizes other
(C) retina athletes’ performances.
(D) lens (C) A young man who is shy becomes the
(E) cornea center of attention at a party given by
friends.
(D) After exams were graded and returned, a
college student looked at his low grade and
decided that the test was unfair and difficult.
(E) After a fight with her boyfriend, a woman
yells at her roommate for sitting in her
favorite chair.
93
GO ON TO THE NEXT PAGE.
-7-
39. Damage to the occipital lobe would most 43. Certain cultures often place more emphasis on
likely affect a person’s collective than on personal achievement. One
result of this may be that people of these cultures
(A) balance
are more likely to develop which type of self-
(B) ability to develop plans
system?
(C) vision
(D) fine motor movements (A) Interdependent
(E) language processing (B) Interrelated
(C) Independent
40. James was born with a condition that makes it (D) Integrated
impossible for him to metabolize certain proteins. (E) Inner-directed
Due to early screening and a special diet, he was
able to avoid developing potentially serious 44. A basic assumption underlying short-term
symptoms. Which of the following disorders does memory is that it is
James have?
(A) limited in capacity
(A) Fetal alcohol syndrome (FAS) (B) unlimited in capacity
(B) Down syndrome (C) long-lasting
(C) Autistic disorder (D) not under conscious control
(D) Attention-deficit/hyperactivity disorder (E) resistant to decay
(ADHD)
(E) Phenylketonuria (PKU)
Set A Set B
41. Stanley Milgram’s classic research on obedience
showed that approximately what percentage of
60 60
participants administered the highest voltage
shock? 56 41
(A) 10% 58 76
(B) 25%
62 35
(C) 40%
(D) 60% 61 65
(E) 85%
59 50
42. After staring at a green, black, and orange
“American flag” for about a minute, an 45. Which of the following is true of the two sets of
individual will see a red, white, and blue scores above?
flag afterimage. Which of the following
(A) Set A has a larger standard deviation.
explains this phenomenon?
(B) Set B has a larger standard deviation.
(A) Trichromatic theory (C) The range is the same for both distributions.
(B) Opponent-process theory (D) Set A has a lower median score than set B.
(C) Retinex theory (E) The mean score is the same for both
(D) Color constancy distributions.
(E) Convergence
94
GO ON TO THE NEXT PAGE.
-8-
46. Carol Gilligan, in her criticism of Lawrence 51. People who are color blind most likely have
Kohlberg, proposed that the moral reasoning deficiencies in their
of males is primarily based on
(A) rods
(A) male repression of females, whereas the (B) cones
moral reasoning of females is based on (C) lens
economics (D) optic nerve
(B) rational abstract principles, whereas the (E) occipital lobe
moral reasoning of females is based on
relationships and the social context 52. Research on human mating preferences suggests
(C) legalistic ideals, whereas the moral reasoning that men place greater value on physical
of females is based on more humanistic attractiveness and youthfulness, whereas women
ideals place greater value on social status and financial
(D) observational learning, whereas the moral resources. Which of the following psychological
reasoning of females is genetically points of view best explains this behavior?
determined for the most part
(A) Collectivistic
(E) physical strength, whereas males start
(B) Individualistic
developing morally later than females but
(C) Psychoanalytic
surpass them soon afterward
(D) Evolutionary
(E) Humanistic
47. Which process transfers information from sensory
memory to short-term memory?
53. If Juan tried to learn a long list of words, he
(A) Attention would be most likely to forget words that
(B) Cognition
(A) appeared early in the list
(C) Differentiation
(B) appeared near the end of the list
(D) Perception
(C) appeared in the middle of the list
(E) Sensation
(D) were very unlike the rest of the words
(E) were randomly dispersed throughout the list
48. When the word “walk” is changed to “walked,”
the suffix “ed” is an example of a
54. Which of the following perspectives argues
(A) morpheme that every person has the potential to become
(B) phoneme self-actualized?
(C) lexicon
(A) Humanistic
(D) syntax
(B) Behavioral
(E) language acquisition device (LAD)
(C) Gestalt
(D) Cognitive
49. The just-world hypothesis would best explain
(E) Psychodynamic
which of the following phenomena?
(A) Blaming the victim 55. An image projected to the left visual field
(B) Cognitive dissonance of a split-brained person will be processed in the
(C) The need for power
(A) left visual cortex
(D) Bystander apathy
(B) right visual cortex
(E) Groupthink
(C) right side of the left retina
(D) left side of the right retina
50. Which of the following is a binocular cue for
(E) sensory cortex
depth perception?
(A) Linear perspective
(B) Texture gradient
(C) Interposition
(D) Retinal disparity
(E) Motion parallax
95
GO ON TO THE NEXT PAGE.
-9-
56. A researcher is training laboratory rats to run a 60. Which of the following assessment tools explores
complex maze. Each time the rats learn a new part individuals’ personalities by asking them to
of the maze, they are rewarded with a pellet of examine a series of inkblots and describe what
food. Within a few hours, the rats have learned the they see in the inkblot?
entire maze. Which of the following did the
(A) Thematic Apperception Test
researcher use to teach the rats the maze?
(B) Rorschach Test
(A) Shaping (C) Minnesota Multiphasic Personality
(B) Generalization Inventory-2
(C) Negative reinforcement (D) Halstead-Reitan Battery
(D) A fixed-interval schedule of reinforcement (E) Bender-Gestalt II
(E) Spontaneous recovery
61. Which of the following neurotransmitters
57. Martha is an undergraduate student who is has been linked to Parkinson’s disease and
interested in pursuing a career in psychology. schizophrenia?
She wants to use her knowledge of psychology
(A) Acetylcholine
to help employees become more productive in
(B) Dopamine
the workplace. Which field of psychology should
(C) Gamma-aminobutyric acid (GABA)
Martha select in graduate school?
(D) Norepinephrine
(A) Physiological (E) Serotonin
(B) Cognitive
(C) Educational 62. According to Sigmund Freud, our sexual
(D) Clinical and aggressive instincts are located primarily
(E) Industrial-organizational in the
(A) frontal lobes
58. Although he finds it to be difficult and not
(B) ego
much fun, Tomas puts in long hours practicing
(C) superego
field hockey in the hope of getting an athletic
(D) id
scholarship to college. This best illustrates the
(E) latent stage
idea of
(A) homeostasis 63. Charles Spearman’s concept of g is most
(B) attribution theory accurately defined as
(C) catharsis
(A) a specific type of performance that is
(D) extrinsic motivation
affected by intelligence
(E) arousal theory
(B) one of seven fundamental abilities that
determine behavior
59. The term group polarization refers to the
(C) a single, underlying intellectual capacity
tendency for
measured by intelligence tests
(A) two opposing factions to emerge within (D) the ability to create novel solutions to
a group complex situations
(B) the prevailing opinion within a group (E) the storehouse of knowledge and facts
to become more extreme after discussion that we accumulate during our adult
(C) a single leader to eventually dominate most years
groups
(D) one or more group members to eventually
have their opinions disregarded
(E) democratic leadership to decrease as the
length of meetings increases
96
GO ON TO THE NEXT PAGE.
-10-
64. Which of the following studies demonstrates 69. While grocery shopping, John heard voices
a cross-sectional research design? that seemed to be narrating his every action.
The voices made statements such as “Now
(A) Testing the first graders, and testing them
he is picking up the bread” and “Now he is
again when they are in third grade
putting the bread in his shopping cart.” No
(B) Testing first graders at the end of the
one else heard the voices. John has heard
school year
voices narrating his behavior on several
(C) Testing first, third, and fifth graders
other occasions. What is John experiencing?
at the beginning of the school year
(D) Observing first graders as they test (A) Illusions
in two different subject areas (B) Delusions
(E) Observing first graders as they interact with (C) Hallucinations
either third graders or fifth graders (D) Hypnosis
(E) Grandiosity
65. Which type of therapy uses free association,
dream interpretation, and analysis of
transference?
Questions 70-72 are based on this scenario.
(A) Cognitive-behavioral
(B) Humanistic
A researcher randomly assigned boys and girls
(C) Behavioral
to each of two groups. One group watched a violent
(D) Psychodynamic
television program while the other group watched
(E) Client-centered
a nonviolent program. The children were then
observed during a period of free play, and the
66. Which of the following psychometric properties is
incidence of aggressive behavior was recorded
used to assess the extent to which the items on an
for each group.
intelligence test measure a person’s intelligence?
(A) Internal consistency
(B) Split-half reliability 70. What is the dependent variable in this study?
(C) Standardization
(A) Sex of the children
(D) Construct validity
(B) Duration of free play
(E) Predictive validity
(C) Type of television program viewed
(D) Level of televised violence
67. Marc performs poorly on a psychology exam and
(E) Incidence of aggressive behavior
explains his failure by saying, “That test was so
hard no one could pass it.” This explanation
71. What is the independent variable in this study?
illustrates
(A) Incidence of aggressive behavior
(A) reciprocal determinism
(B) Type of television program viewed
(B) self-serving bias
(C) Sex of the children
(C) the fundamental attribution error
(D) Number of groups
(D) the representativeness heuristic
(E) Duration of free play
(E) the just-world hypothesis
72. This research method is best
68. Martin fell off his skateboard and badly bruised
characterized as
his elbow. He immediately began rubbing the
area around the bruise until the pain subsided. (A) experimental
This method of reducing pain can be explained (B) correlational
by which of the following? (C) longitudinal
(D) naturalistic observation
(A) Gate-control theory
(E) case study
(B) Opponent-process theory
(C) Trichromatic theory
(D) Expectancy theory
(E) Phantom pain
97
GO ON TO THE NEXT PAGE.
-11-
73. John is completing a lengthy test in which he 77. A teacher finds the distribution of scores on
must indicate whether various written statements a final exam to be positively skewed with low
are true or false about himself. He is most likely variability. On the basis of this information, the
taking which of the following? teacher would be most justified in concluding that
(A) A personality inventory (A) a small number of students in the class
(B) A projective test did poorly on the exam
(C) An achievement test (B) the students in her classroom have a very
(D) An intelligence test wide range of intellectual abilities
(E) A neuropsychological test (C) the exam was too difficult
(D) the exam is not a reliable assessment tool
74. Damage to which of the following brain structures (E) most of her students are of above average
may cause the inability to detect the emotional ability
significance of facial expressions, especially
those demonstrating fear? 78. In which of the following techniques do
researchers inject a harmless, radioactive
(A) Hippocampus
substance into the living human brain to
(B) Thalamus
examine activity?
(C) Cerebellum
(D) Hypothalamus (A) Lesioning
(E) Amygdala (B) Electroencephalogram (EEG)
(C) Computerized axial tomography (CAT)
75. Of the following cognitive milestones, (D) Positron-emission tomography (PET)
which ability tends to be acquired last? (E) Magnetic resonance imaging (MRI)
(A) Telegraphic speech
79. An individual who stomps angrily out of a
(B) Use of schemas
restaurant after being kept waiting five extra
(C) Object permanence
minutes for a reserved table may be exhibiting
(D) Hypothetical thinking
symptoms of
(E) Assimilation
(A) schizophrenia
76. Paul Ekman found that when Japanese students (B) antisocial personality disorder
watched films of surgery, they masked their (C) avoidant personality disorder
expressions of disgust with a smile when (D) dependent personality disorder
an authority figure entered the room but not (E) narcissistic personality disorder
when alone. American students maintained
their expressions of disgust both alone 80. Students who enjoyed solving a puzzle were
and in the presence of an authority figure. rewarded for doing so. Later, they played less
Ekman’s findings illustrate what he calls with the puzzle than did their counterparts who
were not rewarded for the same task. This
(A) the facial feedback hypothesis
illustrates which of the following principles?
(B) display rules
(C) phlegmatic personalities (A) Latent learning
(D) the two-factor theory (B) Self-fulfilling prophecy
(E) adaptation-level phenomenon (C) Intermittent reinforcement
(D) The overjustification effect
(E) The law of effect
98
GO ON TO THE NEXT PAGE.
-12-
81. Jim is better at computer games when his 85. Which procedure can reduce problems associated
friends are watching than when he plays alone. with the use of deception by a researcher?
Researchers would explain Jim’s behavior using
(A) Debriefing
which of the following theories?
(B) Random sampling
(A) Social facilitation (C) Random assignment
(B) Social loafing (D) Using only single-blind studies
(C) Group polarization (E) Using only double-blind studies
(D) Groupthink
(E) Normative social influence 86. Psychologists from which of the following
perspectives of personality are most interested
82. Which of the following theories suggests that in assessing a person’s locus of control?
a physiological need creates a state of tension
(A) Psychoanalytic
that motivates an organism to satisfy the need?
(B) Cognitive
(A) Opponent-process (C) Evolutionary
(B) Drive-reduction (D) Humanistic
(C) Incentive (E) Gestalt
(D) Arousal
(E) Gate-control 87. Elizabeth picks up the clothes from her bedroom
floor and puts them away to avoid her mother’s
83. According to Jean Piaget, children cease to repeated nagging. Elizabeth’s behavior is being
exhibit egocentrism during which of the influenced by
following stages?
(A) classical conditioning
(A) Sensorimotor (B) positive reinforcement
(B) Preoperational (C) negative reinforcement
(C) Concrete operational (D) extinction
(D) Post-conventional (E) punishment
(E) Pre-conventional
88. The diathesis-stress approach would likely
84. The fundamental attribution error occurs when support which of the following statements about
individuals do which of the following? psychological disorders?
(A) Mold their interpretations of the past to fit (A) Disorders are a result of the social and
how events actually turned out economic situations in which people live.
(B) Incorrectly assume that virtually all behavior (B) Disorders are a result of unresolved,
is determined by genetic factors unconscious conflicts between the id and
(C) Underestimate the influence of unconscious the superego.
motivation when trying to explain their own (C) Disorders are a result of negative events that
behavior trigger irrational thoughts.
(D) Overestimate the influence of personal (D) Disorders are a result of whether or not
qualities relative to situational factors when showing emotions is accepted by the
trying to explain the behavior of others individual.
(E) Assume that very attractive people tend to be (E) Disorders are a result of predisposed,
more intelligent and more competent than biological factors triggered by the
people who are somewhat less attractive environment.
99
GO ON TO THE NEXT PAGE.
-13-
89. John regularly stops at the pharmacy to collect 92. When given a drug that produced general arousal,
pamphlets that list symptoms of different research participants placed in a room with a
illnesses, because he is worried about his health. happy confederate described their emotional state
Each day he carefully monitors his vital signs, and as happy, while those placed in a room with an
he also frequently meets with a physician. On his angry confederate described their emotional state
most recent visit, the physician suggested that as angry. Which theory of emotion best explains
John was perfectly healthy. With which of the these results?
following psychological disorders might John be
(A) James-Lange
diagnosed?
(B) Ekman
(A) Somatization disorder (C) Cannon-Bard
(B) Conversion disorder (D) Schachter-Singer
(C) Hypochondriasis (E) Opponent-process
(D) Generalized anxiety disorder
(E) Dissociative disorder 93. Under hypnosis, Jerry is able to withstand pain
without showing any outward signs of discomfort.
90. The Psychology Aptitude Test (PAT) was However, when asked to signal if some part of his
administered to incoming college psychology consciousness is aware of the pain, he raises his
majors. Their scores were later compared to their hand. Which of the following is a theory that best
performance in the introductory psychology explains Jerry’s behavior?
course, and high scores on the PAT were related
(A) Dissociation
to high grades in the course. Therefore, the PAT
(B) State
has
(C) Role
(A) adequate standardization (D) Social influence
(B) internal consistency (E) Age regression
(C) face validity
(D) predictive validity 94. A person eats a hamburger at a restaurant and
(E) content validity develops a very bad stomachache after finishing
eating. As a result of the sudden illness, the
91. Joseph never sleeps through the night. person cannot eat hamburgers anymore. Just
He wakes up at least once per hour to check thinking about them makes the person feel sick
all the doors and windows in his house to make to the stomach. In this scenario, the thought of
sure they are locked and to check the stove a hamburger is
to make sure it is turned off. Joseph’s behavior
(A) an unconditioned stimulus
would be classified as
(B) an unconditioned response
(A) an obsession (C) a conditioned stimulus
(B) an avolition (D) a conditioned response
(C) a fixation (E) a negative reinforcer
(D) a panic attack
(E) a compulsion
GO ON TO THE NEXT100
PAGE.
-14-
95. Research on stereotype threat indicates that 97. Which of the following is the best example
students might not do as well as they can of a homeostatic process?
on a test if
(A) Manny decides that he is overweight
(A) they are informed that people of their and goes on a diet.
ethnicity, age, or gender usually do not (B) Cathy drinks a large amount of water
perform well on the tests to reduce thirst after a long race.
(B) the group taking the test is not ethnically (C) Bert eats nothing but fruits and grains
diverse for a week before a huge holiday dinner.
(C) they are forced to take a test that is known (D) Edesa stays up later than normal to study
to have low test-retest reliability for a test.
(D) other students perceive them to be of (E) Lian becomes angry after sitting in traffic
a minority ethnic group for an hour and a half.
(E) the test does not have standardized
administration or scoring procedures 98. Brad hears a report on the evening news that diets
low in carbohydrates are beneficial to one’s
96. When parents set few controls on their children’s health. Considering this advice, he begins such a
television viewing, allowing the children freedom diet. Later he hears another report condemning
to set individual limits, make few demands, and low-carbohydrate diets as harmful to one’s health.
do not punish for improper television viewing, the Based upon research on belief perseverance, how
parents exemplify a parenting style referred to as would Brad respond to this new information?
(A) permissive (A) Decide to begin a low-calorie diet and
(B) authoritative increase his physical activity
(C) authoritarian (B) Believe the second news story and
(D) rejecting-neglecting discontinue his diet
(E) pessimistic (C) Study low-carbohydrate diets on his own
(D) Continue to believe in the beneficial effects
of low-carbohydrate diets
(E) Decide to try a high-carbohydrate diet instead
GO ON TO THE NEXT101
PAGE.
-15-
Questions 99-100 refer to the illustration below. 99. Which of the following brain regions
is indicated by the arrow?
(A) Cerebellum
(B) Amygdala
(C) Hypothalamus
(D) Hippocampus
(E) Pons
END OF SECTION I
IF YOU FINISH BEFORE TIME IS CALLED, YOU MAY
CHECK YOUR WORK ON THIS SECTION.
102
-16-
Section II
Free-Response Questions
103
-17-
PSYCHOLOGY
SECTION II
Time—50 minutes
1
Percent of total grade—33
3
Directions: You have 50 minutes to answer BOTH of the following questions. It is not enough to answer a question
by merely listing facts. You should present a cogent argument based on your critical analysis of the questions posed,
using appropriate psychological terminology.
1. The goal of a psychology class project is to have students teach a course concept to their classmates as a review
for a later examination. Students in the class have been randomly assigned to work in small groups on the
project. Each group’s grade for the project will be based on how well the entire class performs on the exam
section that tests knowledge of the concepts each group respectively presents.
A. Explain how each of the psychological concepts below may affect the development of student projects.
• Social loafing
• Mental set
• Self-fulfilling prophecy
• Extrinsic motivation
B. Explain how each of the following concepts might influence the class’s ability to retain the information
presented by the groups.
• Proactive interference
• Absolute threshold
GO ON TO THE NEXT104
PAGE.
-18-
2. James was the lead in the school play. It was opening night. Mr. Ramirez, the director, asked James if he was
ready to go on and reminded him about a few last-minute changes in the script.
A number of thoughts and feelings came over James as the curtain went up. James became anxious when he saw
all the faces of the audience members. Then he saw his mother, who is a professional actress, sitting in the front
row. James could not relax.
At that instant, he saw his psychology teacher in the audience. He knew that he needed to use the concepts
learned in her class to make sense of these thoughts, feelings, and surroundings.
Describe how each of the following concepts might help James give a strong performance.
• State-dependent memory
• Arousal theory
• Positive reinforcement
• Selective attention
• Imagery
• Social facilitation
• Sensory adaptation
STOP
END OF EXAM
105
-19-
AP® Psychology
Multiple-Choice Answer Key
106
-21-
AP® Psychology
Free-Response Scoring Guidelines
Question 1
The goal of a psychology class project is to have students teach a course concept to their classmates as a review
for a later examination. Students in the class have been randomly assigned to work in small groups on the project.
Each group’s grade for the project will be based on how well the entire class performs on the exam section that
tests knowledge of the concepts each group respectively presents.
A. Explain how each of the psychological concepts below may affect the development of student projects.
• Social loafing
• Mental set
• Self-fulfilling prophecy
• Extrinsic motivation
B. Explain how each of the following concepts might influence the class’s ability to retain the information
presented by the groups.
• Proactive interference
• Absolute threshold
Scoring Criteria
General Considerations
1. Answers must be presented in sentences, and sentences must be cogent enough for the student’s meaning
to come through. Spelling and grammatical mistakes do not reduce a student’s score, but spelling must be
close enough so that the reader is convinced of the word.
2. Within a point, a student will not be penalized for misinformation unless it directly contradicts correct
information that would otherwise have scored a point.
3. A student can only score points if information is presented in the context of the question. For example, it
must be clear to the reader that the student is discussing social loafing to score Point 1. The best way for a
student to establish context is to explicitly state it (e.g., “Social loafing may affect the development of
student project by…”). In the absence of such language, the reader may infer context if the paragraph
structure or order of the answer makes the context clear. However, if a student uses language like,
“individual effort is often reduced on a group project” without identifying this concept as social loafing,
the point cannot be scored because the reader has no way of knowing for sure that the student understands
that this is social loafing.
4. While students may define the concept in question, definitions alone are never sufficient to score the
point. For all points, students must make clear their understanding of the concept through their analysis of
the application of the concept to the group project example.
107
-22-
AP® Psychology
Free-Response Scoring Guidelines
Question 1 (continued)
PART A. Psychological concepts that may affect the development of the group projects. All of the
psychological concepts in Part A apply to individuals within a group, not to the group as a whole. Do not
award the point unless this is clear.
Point 1: Social loafing occurs when individual effort is reduced on a group project.
• The student must clearly recognize that social loafing would have a negative effect on the group project.
Example: “Social loafing would lead to poorer group projects because individual members of the group
wouldn’t work as hard.”
Point 2: Mental set is the tendency to approach a problem in a specific way, typically in a way that has worked
in the past.
• Mental set could have either a positive or negative effect on the group project.
Example: “The group members assign each member a part of the project to prepare, as they have in the past.
When one group member does not prepare and present his or her part of the project, the entire group gets a
bad grade. Using an approach that had worked in past projects was ineffective in this case.” (This point only
scores if context is established—see general consideration number 3 above.)
Point 3: Self-fulfilling prophecy occurs when a person’s expectation of another person leads that person to
behave in the expected way.
• Self-fulfilling prophecy could have either a positive or negative effect on the group project.
Example: “If the leader of a group expected members of the group to do poorly (or well), the members might
not do as well because of this self-fulfilling prophecy.”
Point 4: Extrinsic motivation occurs when a person has a desire to do something to gain a reward or avoid a
punishment.
• In this case the reward would be the good grade that would result from effectively teaching the concept
assigned to the group. Therefore, extrinsic motivation should have a positive effect on the group project.
Example: “A member of the group might work especially hard because she is extrinsically motivated to earn a
high grade in the class.”
108
-23-
AP® Psychology
Free-Response Scoring Guidelines
Question 1 (continued)
PART B. Psychological concepts that might influence the class’s ability to retain information.
Point 5: Proactive interference occurs when an older memory disrupts the ability to recall a newer memory.
• The material presented by an earlier group might make it more difficult for students to remember the
material presented by a later group. Therefore, proactive interference would have a negative effect on the
group project.
Example: “When they take the exam, students in the class might have trouble remembering what a small
group presented because something they had learned earlier interferes with the information.”
Point 6: Absolute threshold is the minimum stimulus strength necessary for a person to detect a stimulus.
• Students in the class must be aware of a small group’s presentation in order to retain the information from
the presentation. If the presentation was too quiet to hear (or if a PowerPoint presentation was not visible
to someone in the class), the students would not be able to retain the information because they never had
an opportunity to encode it.
Example: “Because it was below the student’s absolute threshold, a student in the back of the class couldn’t
hear a small group’s presentation. This student couldn’t retain information that he never heard.”
109
-24-
AP® Psychology
Free-Response Scoring Guidelines
Question 2
James was the lead in the school play. It was opening night. Mr. Ramirez, the director, asked James if he was
ready to go on and reminded him about a few last-minute changes in the script.
A number of thoughts and feelings came over James as the curtain went up. James became anxious when he saw
all the faces of the audience members. Then he saw his mother, who is a professional actress, sitting in the front
row. James could not relax.
At that instant, he saw his psychology teacher in the audience. He knew that he needed to use the concepts learned
in her class to make sense of these thoughts, feelings, and surroundings.
Describe how each of the following concepts might help James give a strong performance.
• State-dependent memory
• Arousal theory
• Positive reinforcement
• Selective attention
• Imagery
• Social facilitation
• Sensory adaptation
Scoring Criteria
General Considerations
1. Answers must be presented in sentences, and sentences must be cogent enough for the student’s meaning
to come through. Spelling and grammatical mistakes do not reduce a student’s score, but spelling must be
close enough so that the reader is convinced of the word.
2. Within a point, a student will not be penalized for misinformation unless it directly contradicts correct
information that would otherwise have scored a point.
3. A student can only score points if information is presented in the context of the question. For example, it
must be clear to the reader that the student is discussing state-dependent memory to score Point 1. The
best way for a student to establish context is to explicitly state it (e.g., “State-dependent memory might
help James’s performance in the play because…”). In the absence of such language, the reader may infer
context if the paragraph structure or order of the answer makes the context clear. However, if a student
uses language like, “James may perform better if he’s feeling the same during the performance as he did
during rehearsal” without identifying this concept as state-dependent memory, the point cannot be scored
because the reader has no way of knowing for sure that the student understands that this is state-
dependent memory.
4. While students may define the concept in question, definitions alone are never sufficient to score the
point. For all points, students must make clear their understanding of the concept through their analysis of
the application of the concept to the school play scenario.
110
-25-
AP® Psychology
Free-Response Scoring Guidelines
Question 2 (continued)
Point 1: State-dependent memory occurs when recall improves when individuals are in the same psychological
state they were in when they encoded information.
• Here, the student must show how a congruent psychological state (mood, drug state, etc.) during the play
would enhance James’s ability to retain information from the play rehearsal.
Example: “James will do better if he imagines himself in the same emotional state he was in during
rehearsal.” (This point only scores if context is established—see general consideration number 3 above.)
Point 2: Arousal theory. For this point, accept any discussion appropriate to arousal theory. This theory dictates
that optimum performance occurs with moderate levels of arousal.
• The student must relate James’s performance to his arousal level. If he is not sufficiently aroused
(“psyched up”), he should increase his arousal to perform well. If he is too aroused (“freaking out”), he
should decrease his arousal for optimum performance. Since the question indicates that James “became
anxious,” “could not relax,” and “needed to calm his mind,” the student’s answer must recognize the
importance of reducing arousal.
Example: “James sees that he has become too aroused and must get himself calmed down to give his best
performance.”
Point 3: Positive reinforcement is any desired consequence of a behavior that increases the likelihood of that
behavior occurring again in the future.
• The student needs to establish that positive reinforcement for good performance at some point in the past
increases the chance that James will do well on opening night.
• The student may indicate that positive reinforcement for James’s performance this evening may increase
the likelihood that he will participate in a play in the future.
Example: “Mr. Ramirez praises James for a good performance during rehearsal. This positive reinforcement
should increase the likelihood that James will perform well on opening night.”
Point 4: Selective attention is when one focuses on one stimulus to the exclusion of other stimuli.
• The student must identify that there are distractions on opening night that might cause James to make
mistakes, and recognize that he must selectively attend to the cues and his role to perform well.
Example: “James has to block out all of the noise and excitement of opening night and focus on his role. His
ability to selectively attend will help him.”
111
-26-
AP® Psychology
Free-Response Scoring Guidelines
Question 2 (continued)
• The student may explain how this technique can be used by James as a method for encoding his lines into
memory.
• The student may also explain how imagery can be used as a preparation technique for improving
performance.
Example: “James has been picturing what it would be like to take the stage on opening night. He has been
using mental images to rehearse in advance how the performance should go.”
Point 6: Social facilitation occurs when the presence of others enhances performance.
• The student must recognize that James should do well in a performance situation because the audience
would spark social facilitation.
Example: “The presence of the audience on opening night will cause James to perform better than he would
without the audience.” (This point only scores if context is established—see general consideration number 3
above.)
• The student must argue that James can adapt to some type of sensory stimulation (e.g, stage lights,
audience noise) after being exposed to it for some time.
Example: “The stage lights went off at the end of a scene and James could not see in the darkness. After a few
moments, he could see well enough to find his next place on the stage.” (This point only scores if context is
established—see general consideration number 3 above.)
112
-27-
Psychology Practice Exam
This Practice Exam from the 2012 international administration is provided by the College
Board for AP Exam preparation. Teachers are permitted to download the materials and make
copies to use with their students in a classroom setting only. To maintain the security of this
exam, teachers should collect all materials after their administration and keep them in a
secure location.
Exams may not be posted on school or personal websites, nor electronically redistributed for
any reason. Further distribution of these materials outside of the secure College Board site
disadvantages teachers who rely on uncirculated questions for classroom testing. Any
additional distribution is in violation of the College Board’s copyright policies and may result
in the termination of Practice Exam access for your school as well as the removal of access to
other online services such as the AP Teacher Community and Online Score Reports.
113
B
This is the multiple-choice section of the 2012 AP exam. It includes cover material and
other administrative instructions to help familiarize students with the mechanics of
the exam. (Note that future exams may differ in look from the following content.)
114
PSYCHOLOGY
SECTION I
Time— 1 hour and 10 minutes
100 Questions
Directions: Each of the questions or incomplete statements below is followed by five suggested answers or
completions. Select the one that is best in each case and then fill in the corresponding circle on the answer sheet.
1. A psychotherapist who believes that deviant 4. The area of the brain stem that is important in
behavior can be traced either to genetic anom- controlling breathing is the
alies or to problems in the physical structure of
(A) suprachiasmatic nucleus
the brain most likely subscribes to which of the
(B) cerebellum
following views of abnormality?
(C) limbic system
(A) Cognitive (D) medulla
(B) Behavioral (E) hippocampus
(C) Biomedical
(D) Sociological 5. Egocentrism, animism, and artificialism are
(E) Psychoanalytic characteristic of which of Jean Piaget’s stages
of cognitive development?
2. A person with obsessive-compulsive disorder is
(A) Sensorimotor
best described as an individual who experiences
(B) Preoperational
(A) memory loss (C) Postformal
(B) intense mood swings (D) Concrete operations
(C) extreme fear of open spaces (E) Formal operations
(D) physical symptoms with no known cause
(E) persistent anxiety-provoking thoughts 6. Which of the following would an industrial-
organizational psychologist be LEAST likely
3. A research design involves two randomly to study?
assigned groups of participants. One group
(A) Managerial skills
receives a one-time treatment, and the other does
(B) Employee motivation
not. Later, the two groups are compared to see
(C) Job satisfaction
whether the treatment had an effect. Psychologists
(D) Corporate profitability
call this kind of research
(E) Pay incentive programs
(A) a correlational study
(B) an experiment
(C) a case study
(D) a survey
(E) a cross-sequential study
18. Which of the following systems produces, 22. When is it permissible for a psychologist to share
circulates, and regulates levels of hormones in a client’s test scores with another person?
the body?
(A) When an employer inquires about the mental
(A) Circulatory system health status of the client
(B) Endocrine system (B) When the client provides written permission
(C) Limbic system to share results
(D) Sympathetic nervous system (C) When a school official requests the test
(E) Parasympathetic nervous system scores to aid in a college admission decision
(D) When the test scores are within the normal
range
(E) Never
83. Mental shortcuts or rules of thumb that help solve 84. The Minnesota Multiphasic Personality Inventory
problems and reduce mental effort are called is used primarily to provide information about
which of the following?
(A) heuristics
(B) algorithms (A) Achievement
(C) syllogisms (B) Communication skills
(D) propositions (C) Intelligence
(E) concepts (D) Body image
(E) Clinical disorders
86. Which of the following is a partial reinforcement 88. Which of the following concepts provides the best
schedule that is most resistant to extinction? explanation for why people seek to put on warmer
clothing when they start to feel cold?
(A) Noncontingent
(B) Shaping (A) Set-point theory
(C) Variable ratio (B) Homeostasis
(D) Fixed ratio (C) Self-serving bias
(E) Fixed interval (D) Refractory period
(E) Assimilation
87. A young child shown a nine-inch round bowl and
a six-inch round bowl containing equal amounts
of popcorn says he is certain the smaller bowl has
89. The linguistic relativity hypothesis of
more popcorn than the larger bowl. This child has
Benjamin Whorf suggests which of the
yet to acquire what Jean Piaget called
following?
(A) object permanence
(A) People of different cultures use similar
(B) equilibrium
words for common objects.
(C) functional fixedness
(B) Languages with many words to describe
(D) conservation
certain phenomena lack deep structure.
(E) circular reactions
(C) The number of phonemes used in spoken
language is universal across cultures.
(D) People of cultures with few words
to describe certain phenomena are more
precise in their descriptions.
(E) Speakers of different languages think
differently due to the differences in their
languages.
92. To score high on a test of creativity, a person’s 95. Brain damage that leaves a person capable of
answers should be understanding speech but with an impaired ability
to produce speech most likely indicates injury to
(A) original and valuable
which of the following?
(B) normative and divergent
(C) normative and convergent (A) The basal ganglia
(D) unconscious and illogical (B) Wernicke’s area
(E) simple and contextual (C) The substantia nigra
(D) Broca’s area
(E) The inferior colliculi
END OF SECTION I
IF YOU FINISH BEFORE TIME IS CALLED, YOU MAY
CHECK YOUR WORK ON THIS SECTION.
-20-
B
This is the free-response section of the 2012 AP exam. It includes cover material and
other administrative instructions to help familiarize students with the mechanics of
the exam. (Note that future exams may differ in look from the following content.)
132
Question 1 is reprinted for your convenience.
1. Psychological research methodologies and statistics are characterized by strengths and weaknesses in
investigating behavior. Each method or statistic is best suited for certain research questions. For each pair
below, describe a condition under which one is more appropriate than the other.
Mean, median
Descriptive statistics, inferential statistics
Longitudinal study, cross-sectional study
Single-blind technique, double-blind technique
Random assignment, random selection
Survey, case study
Correlational study, experiment
Multiple-Choice Questions 50 E
Question # Key 51 E
1 C 52 E
2 E 53 D
3 B 54 C
4 D 55 D
5 B 56 E
6 D 57 A
7 A 58 C
8 D 59 D
9 B 60 A
10 E 61 E
11 E 62 A
12 D 63 B
13 C 64 C
14 D 65 A
15 D 66 E
16 D 67 C
17 E 68 D
18 B 69 C
19 A 70 C
20 A 71 C
21 D 72 D
22 B 73 B
23 A 74 D
24 C 75 B
25 D 76 C
26 C 77 D
27 C 78 B
28 C 79 E
29 A 80 D
30 E 81 E
31 E 82 C
32 A 83 A
33 C 84 E
34 D 85 B
35 B 86 C
36 A 87 D
37 B 88 B
38 E 89 E
39 C 90 C
40 B 91 A
41 D 92 A
42 B 93 D
43 B 94 B
44 C 95 D
45 D 96 A
46 B 97 C
47 E 98 A
48 E 99 D
49 D 100 B 135
AP® Psychology
2012 SCORING GUIDELINES
INTERNATIONAL OPERATIONAL
Question 1
Psychological research methodologies and statistics are characterized by strengths and weaknesses in
investigating behavior. Each method or statistic is best suited for certain research questions. For each pair
below, describe a condition under which one is more appropriate than the other.
• Mean, median
• Descriptive statistics, inferential statistics
• Longitudinal study, cross-sectional study
• Single-blind technique, double-blind technique
• Random assignment, random selection
• Survey, case study
• Correlational study, experiment
General Considerations
1. The student’s answer MUST establish the context of the question in order to score. For example,
definitions alone will not score. There must be a direct or implied comparison (e.g., “better”) between the
two terms in order to establish “more appropriate.”
2. If the student answers both terms of the pair and gets only one correct, score the point UNLESS the
second answer directly contradicts the first.
Question 2
Lauren loves her parents very much but is so committed to playing the violin that she transferred from her original
college to a music school against their wishes. Lauren practices and studies violin with a group of talented
students. She aspires to a life as a concert musician.
A. Explain how the concept of approach-approach conflict may have contributed to Lauren’s decision
to change schools.
B. Explain how self-actualization may contribute to Lauren’s desire to play the violin.
C. Using the following psychological terms, describe why Lauren plays the violin.
• Intrinsic motivation
• Social learning theory
D. Explain how each of the following can affect Lauren as she plays the violin.
• Acetylcholine
• Difference threshold
• Social facilitation
General Considerations
1. All responses must relate back to the prompt.
2. Students may not use circular definitions to describe terms.
3. Points will not be taken away from a student for incorrect information unless there is a direct
contradiction.
4. Students need not answer the question in order. However, if responses are presented out of order,
students need to create context for the portion of the question they are answering.
5. Definitions alone do not score. However, they may be used as a consideration to support the information
in the application portion of the response.
6. Responses must be presented in complete sentences.
Point 1: Explain how the concept of approach-approach conflict may have contributed to Lauren’s
decision to change schools.
Students must indicate both the presence of a conflict involving two positive goals or options and the indication
that these two options/goals result in conflict or tension.
• Lauren loves her parents and wants to continue to live with them, but she also loves music and wants to
go to the best school in the nation, which is hours away from her parents’ home.
• Lauren feels tension and anxiety over the choice she will need to make between two positive outcomes.
Point 2: Explain how self-actualization may contribute to Lauren’s desire to play the violin.
• If Lauren focuses on becoming the best violinist that she can be she will move towards self-actualization.
• If Lauren practices as hard as she can she is working toward reaching her full potential as a musician.
• Lauren is working toward mastery of the playing of the violin.
(DO NOT Score: Lauren tries hard if there is no indication of trying to be the best that she can be.)
Students must include both the observation of the behavior and the corresponding change in behavior or
cognition to receive this point.
• At her new school, Lauren sees the rewards that others receive for their playing. She follows in their
footsteps and practices after seeing them practice.
• Lauren models the violin playing of her classmates.
• Lauren is vicariously reinforced for playing her violin and therefore is motivated to practice more
intensely.
Point 5: Acetylcholine
Students may address those functions enhanced or aided by adequate levels of acetylcholine or those that are
inhibited by a lack of acetylcholine. However, the direction of the relationship to acetylcholine must be correct.
Pathway A: Adequate Levels
(Involved with: Learning, memory, arousal, attention and movement)
• The neurotransmitter acetylcholine helps Lauren to make the fine movements with her hands, which
allows her to play the proper notes on the strings of her violin.
• It also assists her in remembering how to play the notes when she plays the tune again.
• Acetylcholine can help Lauren stay alert and focused while she plays the violin.
(DO NOT score examples that indicate tasks are easier but do not lead to increased performance.)
Sum = ____________
Weighted
Section II
Score
(Do not round)
Composite Score
140